Criminal Procedure -MBE/Federal

¡Supera tus tareas y exámenes ahora con Quizwiz!

SEARCHES CONDUCTED WITH A WARRANT Fourth Amendment:

"...no Warrants shall issue, but upon probable cause, supported by Oath or Affir- mation, and particularly describing the place to be searched, and the persons or things to be seized."

To punish a gambler who owed him money,a bookie purchased from a demolitions experta small explosive device that he planned to place under the driver's seat of the gambler's automobile. The demolitions expert assuredthe bookie that the explosive would be strong enough to cause severe damage to the legs and ankles of anyone sitting in the front seat of the car, but would under no circumstances be strong enough to kill. The device would go off when the ignition was turned on. On Sunday night, the bookie planted the device in the gambler's car. On Monday, the gambler was sick and did not go to work. That afternoon, the gambler's 15-year-old son came home from school and saw his father's car in the driveway. The son took an extra set of keys from the house to take the car for a joyride, even though the gambler had forbidden his son to drive the car without permission. When the 15-year-old turned on the ignition, the explosive device went off and the boy died instantly. If the bookie is charged with the murder of the gambler's son, what is the jury's verdict likely to be? (A) Guilty, because the bookie's act caused the son's death. (B) Guilty, because he intended to cause serious bodily harm to the gambler. (C) Not guilty, because he did not intend to cause the death of any person. (D) Not guilty, because it was not foreseeable that the son would attempt to drive the car.

(B) The bookie is guilty because his act, which caused the death of the gambler's son, was committed with an intent to cause serious bodily harm. At common law, murder was the killing of a human being with malice aforethought. The mental state of malice aforethought could be established with any one of the following states of mind: (i) intent to kill; (ii) intent to cause serious bodily harm; (iii) reckless indifference to an unjustifiably high risk to human life ("depraved heart"); or (iv) the intent to commit a dangerous felony ("felony murder"). In this question, the bookie clearly intended to cause serious bodily harm to the gambler. Under the doctrine of transferred intent, if the actor intended to kill or cause serious bodily harm to one person and caused the death of another person, malice aforethought as to the unintended victim was established. Thus, under the doctrine, the bookie's intent to harm the gambler will be transferred to the gambler's son, and the bookie will be guilty of the murder of the gambler's son. (A) is not as good an answer as (B). It is true that the bookie's act caused the gambler's son's death and that cause must be established to hold the bookie responsible for murder. However, it will take more than "cause" to hold the bookie criminally liable; there must also be malice aforethought. Therefore, (A) is too broad a state- ment. (C) is wrong because, at common law, a person could be guilty of murder even if he did not intend to cause a death; intent to cause serious bodily harm, depraved heart, or felony murder are sufficient intent for malice aforethought. (D) is not a very good answer. The unforeseeability that the gambler's son would drive the car or get killed would not, in and of itself, be a defense. The unforeseeable result would at best be a factor in deciding whether the bookie's act was the criminal proximate cause of the death, but the facts in the question clearly establish proximate cause.

Effect of an Invalid Arrest:

An unlawful arrest does not prevent Defendant from being subsequently prosecuted, nor is it unlawful for the police to continue detaining Defendant so long as they have probable cause. An unlawful arrest might result in the exclusion at trial of evidence that is the fruit of such an arrest. (See Module 5).

Federal narcotics officers suspected the defendant of growing marijuana in his greenhouse, which was connected to his house. The narcotics officers learned from an anonymous informant that the semi-opaque panes of glass on the greenhouse were being replaced during the night with a newer type of glass that let in more light without an increase in visibility. Without a warrant, the officers flew over the defendant's greenhouse with a pair of "night vision" thermal imaging binoculars supplied by the Department of Defense and not available to the general public. He determined that marijuana was being grown. The officers then went to a magistrate, swore out a warrant, and arrested the defendant. If the defendant moves to suppress any evidence gathered by virtue of the flyover, how should the court rule on the motion. A. Deny it, because the police may conduct flyovers to gather evidence. B. Deny it, because the defendant did not live in the greenhouse. C. Grant it, because the "night-vision" binoculars were not available to the general public. D. Grant it, because the flyover was prompted by a tip from an anonymous informant.

C. The use of thermal imaging binoculars to observe the marijuana where it could not be observed by simply. using the naked eye likely renders the search invalid. To be able to a sort a Fourth Amendment right, a person must have a reasonable expectation of privacy with respect to the place searched or the item seized. There is no such expectation of privacy in objects or places held out to the public or that may be viewed from a public vantage point. Thus, the police may fly over an area to observe it with the naked eye, and even a low flyover by a helicopter to view inside a partially covered building is permissible. This is true even if the area is within the curtilage. However, the police may not use technologically enhanced methods that are not available to the public to search areas (at least as to areas within the curtilage). In the instant case, the police have flown over the defendant's green house at night and used a means of enhancing their vision that is not available to the general public. This enabled them to see what could not have been observed with the naked eye, which likely constitutes an impermissible search [See Kyle v US] (A) is incorrect because it is too broad a statement. The police generally may look into any area, even an area within the home or curtilage, by means of a flyover, so long as they do so from a place the public has access to; however, the police may not use technological enhancements that are not available to the public. In the instant case, the use of thermal imaging makes the police conduct unconstitutional. (B) is incorrect, as it does not properly define the connect of "open fields" doctrine, areas outside the "curtilage" (dwelling house and outbuildings) are subject to police search, as these areas are held out to the public and are unprotected by the Fourth Amendment. (B) ignores the fact that an outbuilding may be deemed part of the curtilage. (In determining whether the building is part of the curtilage, the court will consider the proximity of the building to the dwelling, whether the area is enclosed in the same area as the dwelling, and the steps taken by the resident to protect the building from the view of passerby.) (D) is incorrect because it is irrelevant. The officers are not using the informant to establish probable cause for a search warrant. If the flyover had otherwise been permissible, the fact that it was prompted by an anonymous informant would not matter.

HYPO 5C Defendant was arrested for burglary and asked for and received an appointed attorney. Later, police question him about a murder. Defendant makes damaging statements about the murder. No warnings given. What result?

Confession will be surpressed because invoked right under Miranda and covers all crimes, so different crime later suppressed.

Second most important topic:

Confessions, especially in the context of the Fifth Amendment Miranda doctrine

Misdemeanor Manslaughter

Crimes of Assault/Battery were misdemeanors; like felony murder

EXAMPLE Police unlawfully stop suspect without reasonable suspicion, then discover that he has an outstanding arrest warrant, then arrest him and search him incident to arrest.

Discovery of a valid arrest warrant is an intervening act that attenuates the link between intervening arrest and discovery of evidence.

EXIGENT CIRCUMSTANCES b. Destruction of evidence

Ex.: Police can extract a blood sample or a scraping from under a fingernail.

EXIGENT CIRCUMSTANCES a. Hot pursuit of a fleeing felon

Ex.: Police can pursue a felon into his home. They can arrest Defendant, presuming they have probable cause that Defendant has committed an offense, and any evidence or contraband in plain view can be used in a prosecution.

Most important topic:

Fourth Amendment Search and Seizure

STOP AND FRISK d. "Plain Feel" Doctrine:

If police perform a frisk and reasonably and immediately believe that an item is contraband, the item can be removed and used in a prosecution.

"SPECIAL NEEDS" SEARCHES f. Border searches.

Neither citizens nor non-citizens have Fourth Amendment rights at the border with respect to routine searches of persons and effects, including international mail.

HYPO 6A Stinneford robs a bank, buries the loot under a tree in a public park, and goes home and writes aboutit in his diary. The next day, one group of police illegally break into Stinneford's home, read the diary and discover the location of the loot. At the same time, a second group of police talk to a witness who tells them where he saw Stinneford bury the loot. Will the loot be excluded from evidence at trial?

No, not government actors for diary location

"SPECIAL NEEDS" SEARCHES d. Government employees' desks and files.

Permissible to find evidence of employee misconduct.

"SPECIAL NEEDS" SEARCHES e. Students in public schools.

Permissible to investigate violations of school rules. Searches must be based on reasonable grounds, and the school official must use methods that are reasonably related to the purpose of the search and are not excessively intrusive in light of the age and sex of the student and the nature of the infraction.

AUTOMOBILE STOPS Scope

Scope: For safety reasons, police can order both the driver and any passengers out of the vehicle.

AUTOMOBILE STOPS 1. Rule:

Stopping a car for a traffic violation is a seizure for Fourth Amendment purposes. Police must have reasonable suspicion or probable cause to support such a seizure. 2. When the car is pulled over, both the driver and the passenger are seized.

Additional topics:

Wiretapping and Eavesdropping; Law of Arrest; Pretrial Identification; Grand Juries; Pretrial Proceedings; Trial Rights; Guilty Pleas and Plea Bargaining; Punishment; Double Jeopardy; Fifth Amendment Privilege Against Compelled Self-Incrimination ("Taking the Fifth")

HYPO 2C Police obtain probable cause that defendant robbed a gas station. They learn where he lives and works, but wait three months before arresting him on a public sidewalk, with no warrant. Valid arrest?

Yes because they can take long time and they can arrest in public place without a warrant.

Plain view L D P

a) Legitimately on premises b) Discover contraband or fruits, instrumentalities, or evidence of a crime c) In plain view

When Does Jeopardy Attach? Jury trial—when jury _________ and sworn Bench trial—when first ________ sworn Juvenile proceedings—at ___________ of proceeding

empaneled witness commencement

The Supreme Court has held that a person who was on the premises that were searched to buy illegal drugs does not have a reasonable expectation of privacy in the premises. A claim that a search violated the Fourth Amendment can be raised only by a person who has a ________________ of privacy in the place searched.

reasonable expectation

Spontaneous Utterances: Miranda does not apply when Defendant offers a statement that is not in ________ to interrogation.

response

Use of deadly force is a ___________, and deadly force may not be used unless reasonable under the circumstances.

seizure

A computer programmer sent a computer virus anonymously via e-mail to a business. The programmer believed that the virus would just disable the business' email program for a short time without causing any additional damage, although he was aware that it very infrequently caused widespread damage to the infected computer. However, because of a hidden bug in the business's e-mail program, the virus infected the computer's entire hard drive, eventually rendering it unusable. Not only did the business lose important data, it also had to replace the computer, at a cost of over $1,000. The jurisdiction in which this occurred has a modern criminal code patterned after the Model Penal Code. One of its statutes makes it a criminal offense to "knowingly cause over $200 in damage to another's property." May the programmer be found guilty of violating the statute? (A) No, because the programmer did not know that the virus would cause damage to the computer's hard drive. (B) No, because the programmer did not intend to cause the damage to the computer's hard drive. (C) Yes, because the programmer knew that he was sending a virus to the business's email program. (D) Yes, because the programmer was aware that in a very small percentage of cases the virus could cause widespread damage to a computer system.

(A) The computer programmer cannot be found guilty of violating the statute because he did not know that his act would cause damage to the business's computer that it did. Under the Model Penal Code fault standards adopted by

A man beat his live-in girlfriend and fled. The girlfriend called the police and told them about the beating. She also told them that the man likely fled to his best friend's house. The police quickly obtained a valid arrest warrant for the man and went to the friend's house a few hours after the beating. On arriving, the police noticed that a car registered to the man was parked nearby. They knocked and the friend answered the door. The friend told the police that the man was not there. The police pushed past the friend and began searching for the man. They found the man hiding in a closet and arrested him. On searching the man after his arrest, police found cocaine in a small metal box in the man's pants pocket. The man was charged with assault and possession of cocaine. In a pretrial motion, the man moved to suppress the cocaine, claiming that it was the fruit of an unconstitutional arrest. Should the court grant the motion? (A) No, because the police found the cocaine after executing a valid arrest warrant. (B) No, because the search of the friend's house was valid under the community caretaker exception. (C) Yes, because the police entered the house without a search warrant or consent. (D) Yes, because the small metal box could n

(A) The court should not grant the motion. The police may search a person after making a valid arrest. The arrest here was valid because the police had a valid arrest warrant. While the police should have obtained a search warrant to search for the man in the friend's home, the man does not have standing to complain of the illegal search. One may not raise a violation of another's constitutional rights at a Fourth Amendment suppression hearing. A person generally does not have standing to complain about a warrantless search of another's home unless the home was also his home or he was at least an overnight guest in the home. Here, the facts say that the man lived with his girlfriend, fled to the friend's house, and had been there a few hours. Thus, he lacks standing to complain about the warrantless search. (B) is incorrect. The community caretaker exception to the warrant requirement is not applicable here. The exception applies when police are acting to protect a person from imminent physical harm. Here, the beating had ended several hours before and the police had no reason to believe that anyone was in danger in the house.

A homeowner decided to destroy his home by fire in order to collect the insurance. A neigh- bor's house was located a short distance from the homeowner's home. The homeowner knew that there was a strong wind blowing towards the neighbor's home; while he did not want to burn the neighbor's home, he nevertheless set fire to his own home. The fire department was unable to save the homeowner's house. They did manage to put out the fire moments before it spread to the neighbor's home, which suffered damage from smoke and soot. The jurisdiction's arson statute covers burning one's own dwelling as well as the dwelling of another, but is otherwise unchanged from the common law. If the homeowner is charged with attempted arson of the neighbor's home, is he likely to be found guilty? (A) No, because he did not intend to burn the neighbor's house. (B) No, because the fire was put out before any part of the neighbor's home was burned. (C) Yes, because he intended to burn his own home and took a substantial step toward burning the neighbor's house. (D) Yes, because he acted with malice and took a substantial step toward burning the neigh- bor's house.

(A) The homeowner will be found not guilty because he did not have the requisite mental state. To convict a person for an attempted crime, the prosecution must establish that the defendant had an actual specific intent to cause the harm prohibited by the statute and committed an act beyond mere preparation in furtherance of that intent. Those elements—specific intent and act—are required regardless of the mental state required by the target offense. A person who took a substantial step towards commission of the crime but was only reckless with respect to the target offense could not be found guilty of attempt. The homeowner did not intend to burn the neighbor's home. There- fore, he cannot be guilty of attempted arson of the neighbor's home. (B) is wrong. The fact that the fire was put out before it burned any of the neighbor's home would not preclude a conviction of attempted arson if the elements of attempted arson were otherwise established. (C) is wrong. To be guilty of attempted arson of the neighbor's home, the homeowner must have intended to burn the neighbor's home. The doctrine of transferred intent does not apply to attempt. (D) is wrong. A specific intent to burn the home is required for attempted arson. While malice satisfies the state of mind requirement for the completed crime of arson, it will not suffice for attempt.

A gang member determined that he needed to avenge a recent shooting of his friend by killing a member of the rival gang responsible for the shooting. He drank heavily to build up his courage and then went to the home of the rival with a loaded gun and knocked on the door. When the rival opened the door, the gang member pointed the gun at the rival and pulled the trigger. However, due to being intoxicated, he had forgotten to release the safety, so the gun did not discharge. He was easily disarmed by the rival and arrested by the police. At his trial, he testified that he was so intoxicated that he did not remember anything that happened at the rival's house. The crimes below are listed in descending order of seriousness. If the gang member's testimony is believed, what is the most serious crime of which the defendant may be convicted? (A) Attempted murder. (B) Attempted manslaughter. (C) Assault. (D) No crime.

(A) The most serious crime the gang member may be convicted of is attempted murder. At common law, murder is the killing of another human being with malice aforethought. Malice could be shown by the defendant's (i) intent to kill; (ii) intent to inflict great bodily injury; (iii) reckless indifference to an unjustifiably high risk to human life; or (iv) intent to commit a felony. Attempt requires the intent to commit the target crime plus an overt act in furtherance of such intent. Thus, even though murder is a malice crime at common law, attempted murder is a specific intent crime. As a result, voluntary intoxication, which is a defense to specific intent crimes, ordinarily will be a defense to any attempted murder charge. However, one who formed an intent to commit a crime and then drinks in order to work up his nerve to commit it cannot rely on the defense of intoxica- tion, even though he may be too intoxicated to form that intent at the time he did the act. Here, the gang member intended to kill his rival, went to his house with a gun, and pulled the trigger when his rival opened the door. Even if the jury believes his testimony that he did not remember anything that happened at the house, the fact that the defendant had the intent to kill originally and drank to build up his courage negates intoxication as a possible defense. Without such a defense, the defendant may be convicted of attempted murder. (B) is incorrect. Manslaughter requires either a killing committed during the heat of passion or a killing committed by criminal negligence. There are no facts indicating that the defendant was acting under the heat of passion. Furthermore, one cannot be convicted of attempted manslaughter based on negligence theory, because one logically cannot have the specific intent to be negligent. (C) is also incorrect. Although the defendant almost certainly put his rival in fear of an imminent unlawful battery or took sufficient steps to be guilty of the "attempted battery" type of assault, attempted murder would be the far more serious charge of which the defendant could be convicted. (D) is incor- rect because there are at least two crimes of which the defendant could be convicted—attempted murder and assault.

An irate woman seeking vengeance against an appliance store owner for selling her a defec- tive television set went to the store armed with two pistols. Just as she was about to enter the store, she spotted a young mother walking by with a baby carriage. The woman pointed one of the pistols at the mother and one at the carriage, saying, "I hate the owner of this store! Takethis gun, go into his store, and do the job or the kid gets it!" The mother began pleading for her baby's life, but the woman cocked the pistol pointing at the carriage and began counting. The distraught mother took the other pistol and "did the job." Criminal charges are filed against the mother. At trial, she asserts duress as a defense but is found guilty. What is "the job" she did on the store owner likely to have been? (A) "Pistol whipping" the owner. (B) Robbing the owner at gunpoint. (C) Fatally wounding the owner with the pistol. (D) Kidnapping the owner at gunpoint.

(C) The mother will be found guilty if she fatally wounded the owner with the pistol. The defense of duress requires the commission of an otherwise criminal act under the threat of imminent infliction of death or great bodily harm. Duress is a defense to all crimes except homicide. Note, though, that under certain circumstances it may reduce the homicide from murder to manslaughter. The crimes in (A), (B), and (D) would be offenses for which the mother could assert the defense of duress. However, even under duress, she would be found guilty of the homicide in choice (C).

A police officer patrolling at 2 a.m. heard a gunshot and saw a figure leap over a hedge in front of a condominium. Although it was dark, the officer could see that he was carrying a briefcase in one hand and a gun in the other.Just then, someone limped out of the building and pointed at the figure, yelling, "He shot me!" The officer then twice commanded the figure to stop and fired three warning shots in the air. The person kept running, so the officer fired a fourth shot, bringing the person down. The person was later identified as a robber. If the officer is charged with battery, what is the likely verdict of the jury? (A) Not guilty, because he had a right to use deadly force as necessary to stop a fleeing felon posing a threat of serious bodily harm to others. (B) Not guilty, because a police officer cannot commit a crime while acting within the scope of his duty. (C) Guilty, because a police officer does not have the right to use deadly force to prevent a getaway. (D) Guilty, because he did not see the robber steal anything.

(A) The officer should be found not guilty of battery. Police officers are generally entitled to use whatever force is reasonably required, including deadly force, to apprehend or prevent the escape of a felon who poses a threat of serious bodily harm to the officer or others. Here, the robber had apparently just shot someone and was escaping with his gun, justifying the use of deadly force. (B) is an incorrect statement of law; there is no blanket immunity. (C) is wrong because an officer may use deadly force to prevent a getaway if the felon poses a significant threat of death or serious bodily injury to others. (D) is wrong because it is irrelevant whether the officer saw the robber steal anything. He saw the weapon and the wounded victim, and thus had probable cause to believe that the robber was dangerous.

A drug smuggler had just returned home after smuggling in a large quantity of cocaine in the false bottom of his suitcase. As he was about to leave his house again to deliver the cocaine to his contact in the city, a police officer arrived with a trained drug-sniffing dog and asked him if he could come in and ask him some questions. The smuggler declined but the officer stepped into the doorway, and the dog immediately caught the scent of the cocaine and pulled the officer toward the suitcase in the hallway. Based on the dog's clear indication that the suitcase contained narcotics, the police officer opened the suitcase and found the cocaine. The smuggler was then arrested and the cocaine and suitcase seized. At a pretrial hearing, should the judge grant the smuggler's motion to suppress evidence of the cocaine in the suitcase? (A) Yes, because a warrantless search and seizure of items within the defendant's home is not permissible, absent exigent circumstances. (B) Yes, because the search and seizure required a warrant. (C) No, because the cocaine was seized as a search incident to a lawful arrest. (D) No, because there is no reasonable expectation of privacy in the smell of one's suitcase.

(B) The motion should be granted because the search and seizure required a warrant. To have a Fourth Amendment right, a person must have a reasonable expectation of privacy with respectto the place searched or the item seized. In the instant case, the place searched was the defen- dant's home, which clearly is a place in which a person has a reasonable expectation of privacy. Even the entry to a home is within the curtilage and protected against unreasonable searches. Consent to enter was not granted and no other exceptions apply, so evidence of the cocaine should be suppressed. (A) is incorrect because it is too broad of a statement. A warrantless search and seizure of items in one's home may be based on other grounds, such as consent or plain view.(C) is incorrect because the search occurred prior to the arrest and thus was not a search incident to a lawful arrest (even assuming the arrest was lawful). (D) is incorrect. Although it has been held that one does not have a privacy interest in the smell of one's luggage, here the luggage was located in the defendant's home. The fact that the cocaine was still in the defendant's luggage, rather than in some other location within the house, is irrelevant. The search occurred within the defendant's home, which was not permissible under the facts.

During a heated argument, a man punched his female coworker in the stomach after learning that she had been awarded "Employee of the Month." Angered, the woman responded by stabbing the man with a letter opener, which severed his aorta and caused his death. The police arrested the woman. After receiving her Miranda rights, she confessed to killing her coworker, but stated that she had not previously held any grudge against him. When asked why she stabbed him, she stated, "He just made me so mad when he said that I wasa terrible employee and that he deserved the reward instead of me, and I just lost it when he punched me." The district attorney charged the woman with homicide. If the jury believes the woman's statement, of what crime is she most likely to be found guilty? (A) Murder. (B) Voluntary manslaughter. (C) Involuntary manslaughter. (D) Nohomicidecrime.

(B) The woman would most likely be found guilty of voluntary manslaughter in light of the provoking event. Adequate provocation will reduce a killing to voluntary manslaughter if the defendant was both reasonably provoked and actually provoked. The woman also would have to offer evidence that the insults and taunting, followed by a sudden punch in the stomach, would cause an ordinary person to be provoked, which is likely to be the case. Hence, her statement in conjunction with the facts makes this the best answer. (A) is incorrect. At common law, murder was the unlawful killing of a human being with malice aforethought. Malice aforethought could be established with any one of the following states of mind: (i) intent to kill; (ii) intent to cause serious bodily harm; (iii) the depraved heart killing (a reckless indifference to an unjustifiably high risk to human life); or (iv) the commission of a felony. While at least an intent to cause serious harm may be present here, the provocation will reduce the homicide to voluntary manslaughter. (C) is incorrect because involuntary manslaughter is a killing that results from criminal negligence, which theory these facts do not support. (D) is incorrect because, as discussed above, the woman would likely be guilty of voluntary manslaughter.

Three men agreed to rob a restaurant in a nearby town and bought pistols, ski masks, and gloves for that purpose. Prior to entering the restaurant, the man who was to be the lookout had a change of heart and wanted to call offthe robbery. The other two men refused sothe lookout threw down his gun and went tothe nearby bus station to catch a bus back tohis home. The remaining two men went intothe restaurant and robbed it and its patrons. A patrol car happened by as they were leaving,and one of the men seized one of the customers as a hostage. In the exchange of gunfire withthe police, a police officer and the hostage were killed. Both men escaped initially, but oneof them was later captured and charged with robbery and murder. However, because of illegal police conduct in connection with his arrest and subsequent confession, all evidence connecting him with the charged crimes was suppressed and the charges dismissed. If the lookout is also arrested and charged with murder, which of the following is his stron- gest defense? (A) He did not have the requisite mental state to be convicted of murder. (B) He did not physically participate in the events that led to the killings of the police officer and the hostage. (C) He had withdrawn from the plan before the two others began the robbery that led to the killings. (D) He cannot be prosecuted for a crime as to which the principals have not been convicted.

(C) The lookout will not be guilty of murder if he withdrew from the plan before the robbery and killings took place. At common law, each person who took part in the planning of a crime was criminally liable for the crime of conspiracy and for each offense committed in furtherance of the conspiracy. However, if one of the conspirators "withdrew" from the criminal effort before the substantive crimes occurred, he was not liable for the subsequent crimes. To successfully withdraw, the actor must notify all members of the conspiracy that he has withdrawn; this must be done in time for them to have an opportunity to abandon the planned crimes. The facts in the question clearly indicate that he had withdrawn. (A) is not as good an answer as (C) because, if the lookout had not withdrawn, he would be guilty of murder under a felony murder theory.His strongest argument is that he withdrew. (B) is wrong because it is no defense to a chargeof murder that the actor did not physically participate. The lookout would be guilty if he hadnot withdrawn, even without physical participation. (D) is wrong because, if the theory of the prosecutor's murder charge was that the lookout was a conspirator and is liable for all crimes committed in furtherance of the conspiracy, it would make no difference whether the other conspirators are being prosecuted. (All persons must be acquitted for this defense to be effective.)

Several students at a public high schooltold a teacher that a fellow student was selling illegal drugs to other students at the school. The accused student was called into the principal's office and informed of the accusations. The student denied everything, but the principal grabbed the student's purse, which was on top of his desk, and opened it. He removed five small transparent plastic bags, each of which contained a white powder, and immediately called the police. The police arrested the student and conducted tests confirming that the white powder was cocaine. The student was charged with possession of a controlled substance with intent to sell. At her trial, the state planned to introduce the bags and their contents into evidence. The student's attorney moved to suppress the evidence. How should the court rule? (A) Grant the motion, because there was ad- equate time to obtain a warrant. (B) Grant the motion, because the principal lacked probable cause. (C) Deny the motion, because the principal had a reasonable suspicion that the student was selling drugs. (D) Deny the motion, because a principal, acting in loco parentis, has the right to search students whenever he chooses to do so.

(C) The motion should be denied. Due to the nature of the school environment, reasonable grounds for a search are a sufficient basis to justify searches by public school officials. Neither a warrant nor probable cause is required. A school search will be held reasonable if: (i) it offers a moderate chance of finding evidence of wrongdoing; (ii) the measures adopted to carry out the search are reasonably related to the objectives of the search; and (iii) the search is not excessively intru- sive in light of the age and sex of the student and the nature of the infraction. Here, even though the principal did not have probable cause to believe that the drugs were in the student's purse, the principal did have sufficient reasonable grounds to search her purse. Therefore, (A) and (B) are incorrect. (D) is incorrect because it is an overbroad statement of the rights of public school officials.

Suspecting that a husband had slain his wife, police detectives persuaded one of the husband's colleagues at work to remove a drinking glass from the husband's office so that it could be used for fingerprint comparisons with a knife found near the body. The fingerprints matched. The prosecutor announced that he would present comparisons and evidence to the grand jury. The husband's lawyer immediately filed a motion to suppress the evidence of the fingerprint compari- sons so as to bar its consideration by the grand jury, contending that the evidence was illegally acquired. Should the motion be granted? (A) Yes, because, if there was no probable cause, the grand jury should not consider the evidence. (B) Yes, because the employee was acting as a police agent and his seizure of the glass without a warrant was unconstitutional. (C) No, because motions based on the exclu- sionary rule are premature in grand jury proceedings. (D) No, because the glass was removed from the husband's possession by a private citizen and not a police officer.

(C) The motion should be denied. The issue in this question is not whether the seizure of the glass was valid, but whether it is an appropriate time to raise this issue. The exclusionary rule does not apply in grand jury proceedings. According to United States v. Calandra (1974), illegally seized evidence is admissible in grand jury proceedings. A pretrial motion to suppress is the appropriate vehicle to test the constitutionality of the seizure. The grand jury is not the appropriate forum. (A) is incorrect because the function of the grand jury is to consider evidence to determine whether there is probable cause to indict. It is not the function of the judge to take this matter away from the grand jury. A pretrial motion is the appropriate vehicle to challenge probable cause. (B) is incorrect because this choice goes to the legality of the seizure, which is not the issue in this question. The issue in this question is not whether the seizure of the glass was valid, but whether it is the appropriate time to raise the legality of the seizure. (D) is incorrect because it reaches the right result for the wrong reason. This choice goes to the legality of the seizure, which is not the issue in this question, as discussed in the explanation to (B).

A police officer received an anonymous tip that the defendant was at a particular street corner selling drugs. Based solely on the tip,the officer went to the corner, saw the defendant talking with someone, and immediately stopped and frisked the defendant. The officer found an illegal gun as a result of the stop and frisk and arrested the defendant. Prior to trial, the defendant moves to suppress the gun. Should the motion be granted? (A) No, because the anonymous tip gave the officer probable cause to stop and frisk the defendant. (B) No, because the defendant would have been arrested after completing a drug sale, so it was inevitable that the gun would be discovered. (C) Yes, because an anonymous tip, without more, is insufficient to stop and frisk a suspect. (D) Yes, because Miranda warnings were not given.

(C) The motion to suppress the gun should be granted. Police have the authority to briefly detain a person for investigative purposes even if they lack probable cause to arrest. To make such a stop, police must have a reasonable suspicion supported by articulable facts of criminal activity or involvement in a completed crime. When the source of suspicion of criminal activity is an informant's tip, the tip must be accompanied by indicia of reliability sufficient to make the officer's stop reasonable. Here, the stop was based solely on an informant's tip. Because the tip was anonymous and included nothing more than an accusation that a person standing on a certain street corner was selling drugs, it was not sufficient to justify the search. It needed to include more detail (e.g., predicting incriminating movement) to corroborate the accusation. Thus, (A) is incorrect, and the gun should be suppressed under the exclusionary rule as fruit of the poisonous tree. (B) is incorrect. Under the inevitable discovery exception to the exclusionary rule, evidence may be admissible if the police inevitably would have discovered the evidence whether or not they acted unconstitutionally. However, here it is just speculation that the officer would have had grounds to arrest the defendant; this is not sufficient to apply that exception. (D) is incorrect because Miranda warnings are required for custodial police interrogation, not before a stop and frisk.

A suspect was arrested on a charge of bank robbery. After formal charges had been filed, the suspect was scheduled for a lineup identification procedure. The suspect's attorney was notifiedof the lineup and arrived at the station prior to the scheduled lineup. He was directed to wait in the hallway outside the lineup room. When the suspect was escorted into the lineup room, the officer acknowledged the attorney but did not motion for him to follow them into the lineup room. The attorney remained seated in the hallway. Inside the lineup room, the six members of the lineup, including the suspect, stood onone side of the one-way mirror in full view of the witnesses gathered on the other side in the viewing area. The lineup members stepped forward one by one for a closer inspection. After the first two lineup members had been presented, an officer realized that the suspect's attorney was not present and immediately escorted himto the viewing area. The attorney arrived intime to witness the presentation of the suspect and the remaining three members of the lineup. Two witnesses then identified the suspect as the robber. At trial, if the defense objects to the introduction of the lineup evidence, should the objection be granted? (A) No, because the attorney's absence from the lineup procedure was the product of a good faith mistake. (B) No, because the attorney was present when the suspect was presented in the lineup for identification by the witnesses. (C) Yes, because the right to have counsel present at a post-charge lineup includes the right to have counsel present for the entire lineup. (D) Yes, unless the witnesses are unavailable at trial.

(C) The objection should be granted. A post-charge lineup is a critical stage of the prosecution at which a defendant has the right to counsel. Once the government has initiated adversary judicial criminal proceedings, the presence of counsel is a prerequisite to the conduct of a lineup. This right attaches as soon as the accused is within sight of a potential identification witness. Here,the attorney was not present in the room until after the lineup had commenced. (A) is incorrect because the requirement that counsel be present during a post-charge lineup has never turned on a showing of government bad faith, or even government error. The government has an affirma- tive obligation to ensure counsel's presence at a post-charge lineup. (B) is incorrect because the fact that the attorney was present when the suspect was asked to step forward does not remedy the constitutional violation. The accused is entitled to have counsel present at all times during the lineup procedure when the accused is visible to the witnesses. Because the suspect was visible at all times during the lineup, not just when he stepped forward for closer viewing, he was entitled to counsel at all times during the lineup procedure. (D) is incorrect. It is true that, if the witnesses are unavailable at trial, their "prior identification" testimony may be admissible under the Federal Rules of Evidence. However, the evidence is inadmissible regardless because of a violation of the right to counsel.

Standing - Key Question: Does defendant have a reasonable expectation of privacy in the area searched? Under the 4th Amendment, a defendant only has the right to challenge the lawfulness of an invasion of his or her rights, not the rights of a third party. This means that the fact that unlawfully obtained evidence is used against a defendant at trial is not enough in and of itself to give him standing to raise a 4th Amendment challenge. Do individuals have a reasonable expectation of privacy in: a. premises that they own? _____ b. premises they don't own but in which they reside? _____ c. premises in which they are overnight guests? _____. d. another person's residence they are using briefly and solely for business purposes? _____ e. they own the property that was seized? ______________ f. they are passengers in a car that is searched? _________ NOTE: Passenger's "effects" are considered part of the vehicle during an automobile search.

(a) Yes. (b) Yes. (c) Yes (d) No. (e) Only if they have a reasonable expectation of privacy in the area from which it was seized. (f) Only if they have a reasonable expectation of privacy in the area from which it is seized: 1. they own the car; 2. they are the driver; or 3. the item is seized from their person.

ARRESTS 1. When Arrest Occurs: An arrest occurs when Defendant is taken into custody for prosecution or interrogation. 2. Probable Cause: An arrest must be made on the basis of probable cause. There must be trustworthy facts or knowledge sufficient for a reasonable per- son to believe that the suspect has committed or is committing a crime. a. Use of hearsay permissible? ______ b. Can the police arrest a person based on probable cause that she committed an offense that can only result in a fine?____

(a) yes (b) yes

THE FIFTH AMENDMENT AND MIRANDA Police violate a Defendant's right to not "be compelled to be a witness against himself," interpreted to mean a self-incriminating statement, under certain circumstances. 1. The Miranda warnings: a. Right to remain silent: b. Anything you say _____ be used against you; c. Right to an attorney; d. If you can't afford an attorney, the state will provide one _____ of charge 2. The warnings are prophylatic - protect against risk of ________. 3. The warnings are required when we find two things: a. custody; _____ b. interrogation

1(b) will 1(d) free 2. coercion 3. AND

Automobile stops RS EX:RB

1) Generally must have at least reasonable suspicion that a law has been violated 2) Exception—special law enforcement needs can justify suspicionless roadblocks a) Cars must be stopped on basis of a neutral, articulable standard b) Must serve purpose closely related to a particular problem pertaining to automobiles and their mobility

Exceptions to Fruit of Poisonous Tree Exceptions IS A D L IC VN GF IM

1) Independent source—evidence will be admitted if from a source independent of the unconstitutional conduct 2) Attenuation—intervening act or circumstance 3) Inevitable discovery by police 4) Live witness testimony 5) In-court identification 6) Violations of no-knock entry rule 7) Good faith reliance on a defective search warrant 8) Use of evidence to impeach

Investigatory detentions (Terry stops) RSAF RSTC IIR PDR

1) May be made on reasonable suspicion supported by articulable facts 2) Reasonable suspicion determined by totality of circumstances 3) Informer's tips must be accompanied by indicia of reliability 4) Police must act in a diligent and reasonable manner in confirming or dispelling their suspicion (cannot take too long)

SIXTH AMENDMENT RIGHT TO COUNSEL Police violate the right to "assistance of counsel" when they secure a confession from Defendant under certain circumstances. 1. Triggered only at "critical stage" - time when defendant is formally charged with ________. 2. Triggered only if government _______ interrogates the defendant. 3. Caveat: the right is crime specific. Standard is the same as ________ jeopardy (see below). Two crimes are considered different if each crime has an element the other does not. 4. Caveat: Defendant must _______ request the right to counsel. 5. If violation occurs, the confession is barred from case-in-chief but can be used to _______ the defendant at trial.

1. crime 2. agent 3. double 4. expressly 5. impeach

IS THERE A SEARCH WITHIN THE MEANING OF THE FOURTH AMENDMENT? Four questions: 1. Was search done by a __________________? 2. Did the police intrude upon someone's __________________ in the place searched or item seized? 3. Have the police ______________ on a constitutionally protected area __________________? 4. Does ___________ have a reasonable expectation of privacy in the area searched (and thus standing to challenge the search)?

1. government agent 2. reasonable expectation of privacy 3. physically intruded; for the purpose of obtaining information 4. defendant

WHEN A SEIZURE OF THE PERSON OCCURS A seizure of the person occurs when a reasonable person would not feel free to terminate the encounter. There are two bases for a seizure:

1. physical restraint 2. show of authority plus submission (stop in the name of law and you stop = seizure. cops say stop in name of law and you don't stop, not a seizure) *Police are free to conduct consensual interviews, without any belief of criminal activity whatsoever.

FOURTEENTH AMENDMENT "INVOLUNTARINESS" TEST Police violate "due process" when they secure involuntary confessions from Defendants. 1. Assessed on basis of ____________. Factors include suspect's age, education, and experience with system, and manner of police interrogation. Ex.: Police obtain confession from Defendant under threat of whipping. 2. There must be ____________ coercion. Ex.: Merely because a confessing suspect is mentally ill does not dictate that the confession itself was involuntary. Defendant having auditory hallucinations and god telling him turn himself in or kill himself, so went to police station and confessed to murder.

1. voluntariness 2. government

Hijackers of a truck carrying dog food were arrested and indicted by a grand jury. During the grand jury's investigation, the district attorney's office subpoenaed the truck driver as a witness. To her relief, she was not asked questions about a series of thefts of shipments of dog chew-toys that she was involved in, but believes that the subject might be brought up at trial by the defense lawyers. She is afraid that she likely will be fired if she invokes the Fifth Amendment privilege against self-incrimination on the witness stand, so she wants to avoid testifying at all at the trial, even if she is subpoenaed by the defense. As her lawyer, what advice should you give her about complying with a subpoena? A) She cannot avoid testifying, because she can plead the Fifth Amendment on the stand. B) She cannot avoid testifying, and she must answer all questions truthfully, even if her answers will incriminate her, because she is not on trial. C) She can avoid testifying, because she is not a party to the action. D) She can avoid testifying, because the potential that she will incriminate herself is greater than the interest the defense has in calling her as a witness.

A B. applies all the time C. nope D. uh, fake standard

HYPO 2B Defendant is arrested for driving while not wearing a seatbelt, a violation that can result only in assessment of a fine. Valid arrest?

Yes because police have authority when they have probable cause.

At the defendant's prosecution for robbery of a drugstore, the main prosecution witness testified that the defendant had asked her to drive him to the town where the drugstore was located. The witness testified that the defendant did not explain his purpose for going to the town, and that he had stopped at a relative's house along the way to pick up a bundle that could have been the sawed-off shotgun used by the robber. On cross-examination, the defendant's attorney asked a number of pointed questions of the witness, implying that the defendant had asked her to drive to the town so that he could visit relatives there and suggesting that the witness had obtained a sawed-off shotgun for use by a confederate. The defendant did not testify on his own behalf. In final argument, the prosecutor called the jury's attention to the two versions of events suggested by the witness's testimony on direct examination and the defense attorney's questions on cross-examination, and then said, "Remember, you only heard one of the two people testify who know what really happened that day." If the defendant is convicted of robbery, will his conviction likely be upheld? A) No, because the prosecutor's comment referred to the defendant's failure to testify, a violation of his Fifth Amendment privilege of silence. B) No, because under the circumstances the attack on the witness's credibility was not strong enough to permit the prosecutor to mention the defendant's failure to testify in rebuttal. C) Yes, because the prosecutor is entitled to comment on the state of the evidence. D) Yes, because even if it was error to comment on the defendant's failure to testify, the error was harmless beyond a reasonable doubt.

A The defendant's conviction will likely not be upheld because the prosecutor's comment improperly burdened the defendant's assertion of his privilege against self-incrimination. The prosecution is not allowed to comment on the defendant's failure to testify at trial, because the defendant is privileged under the Fifth Amendment to remain silent. (B) is incorrect because no amount of attacks on the credibility of prosecution witnesses will justify such a comment as a rebuttal. (C) is incorrect because the Fifth Amendment privilege outweighs the prosecutor's right to comment on the state of the evidence. (D) is not the best answer even though the harmless error test does apply to improper comments by the prosecution (i.e., the conviction will not be overturned if the prosecution can show beyond a reasonable doubt that the comments did not affect the outcome of the case). Because there is no real indication as to the strength of the case against the defendant, it is impossible to conclude that the error was harmless beyond a reasonable doubt.

May a police officer conduct a warrantless search of a vehicle after validly arresting a vehicle occupant? A Yes, if the officer has reason to believe that the vehicle harbors evidence related to the crime of arrest B Yes, as long as the arrest was valid and the search is contemporaneous with the arrest C No, the car must be impounded and then searched pursuant to established police department policy D No, if the arrestee has already been secured in the police officer's squad car

A A police officer may conduct a warrantless search of a vehicle after validly arresting a vehicle occupant if the officer has reason to believe that the vehicle harbors evidence related to the crime of arrest. The Fourth Amendment prohibits unreasonable searches and seizures. Generally, for a search to be valid, it must be pursuant to a warrant issued by a neutral and detached magistrate and based on probable cause to believe that seizable evidence or fruits of a crime will be found in the place to be searched. Searches incident to arrest are an exception to the general rule. They are allowed without a warrant and may extend beyond the arrestee's person to his "grab area" or "wingspan" (i.e., areas within his reach at the time of arrest). However, when the person arrested was in a vehicle, a search of the vehicle is limited to situations where (i) the arrestee (or more likely arrestees) is (are) unsecured, or (ii) where police have reason to believe that the automobile harbors evidence of the crime for which the arrest was made. Given the above, it is not true that the car must be impounded and then searched pursuant to established police department policy. The police may make a warrantless search at the time and place of arrest if the two exceptions above apply. (Note that it is true that an impounded car can also be searched without a warrant if the search is pursuant to established police department policy.) It is true that a search incident to arrest must be contemporaneous with the arrest. However, it is not true that a search of an automobile is valid as long as the arrest was valid and the search it is contemporaneous and incident to an arrest. Such searches are limited as discussed above (e.g., the arrestee is unsecured or the officer believes the car contains evidence of the crime of the arrest). Finally, it is not true that just because the arrestee is secured in the police officer's squad car that a search cannot be performed. A search of the vehicle incident to arrest can be performed—even if the arrestee is secured—if the officer has reason to believe that the vehicle harbors evidence related to the crime of arrest.

An intervening act will shield the defendant from liability if the act is _______ or _______. A A mere coincidence; outside the foreseeable sphere of risk created by the defendant's act B A criminal act committed by another person; outside the foreseeable sphere of risk created by the defendant's act C Committed by another person who acts intentionally; a mere coincidence

A As a general rule, an intervening act will shield the defendant from liability if the act is a mere coincidence or is outside the foreseeable sphere of risk created by the defendant's act. An intervening act may be the result of an act of nature; it is not necessary that it be committed by another person. Furthermore, an intervening act by another person need not be criminal, nor does it need to be intentionally committed.

Which of the following statements is not correct concerning border searches for illegal aliens? A A roving patrol inside the U.S. border may conduct a warrantless search of an automobile if the officer reasonably suspects that the automobile may contain illegal aliens. B A roving patrol inside the U.S. border may stop an automobile for questioning of the occupants if the officer reasonably suspects that the automobile may contain illegal aliens. C Border officials may stop an automobile at a fixed checkpoint at the U.S. border for questioning of the occupants even without a reasonable suspicion that the automobile contains illegal aliens.

A It is not correct that a roving patrol inside the U.S. border may conduct a warrantless search of an automobile. If the search is not at the border or its functional equivalent, government agents may not perform a warrantless search absent an exception to the warrant requirement (e.g., consent). It is correct that a border search may be made at the functional equivalent of the border, such as where several routes leading directly to the border merge. Border officials may stop an automobile at a fixed checkpoint at the U.S. border for questioning of the occupants even without a reasonable suspicion that the automobile contains illegal aliens. Officials may even disassemble stopped vehicles at such checkpoints, even without reasonable suspicion. There is a diminished expectation of privacy at the border and its functional equivalent due to interests of national sovereignty. Searches there do not require a warrant, probable cause, or even reasonable suspicion. It is true that a roving patrol inside the U.S. border may stop an automobile for questioning of the occupants if the officer reasonably suspects that the automobile may contain illegal aliens. This is a just an application of Terry v. Ohio: A police officer may stop a vehicle when he has articulable reasonable suspicion that a law has been broken. Note, however, that the apparent Mexican ancestry of the automobile's occupants alone is not enough to constitute reasonable suspicion.

The defendant is charged with the battery of a bouncer at a local tavern. At the trial, the prosecutor introduces evidence that while the bouncer was attempting to question the defendant about her intoxicated demeanor, the defendant committed a battery on the bouncer. The defendant attempts to defend against the charge on the basis of self-defense, insisting that the bouncer used excessive force in stopping her from entering the tavern. The defendant attempts to introduce into evidence an authenticated copy of the tavern records that show that three patrons had written complaints against the bouncer within the past six months for the use of excessive force. The prosecutor objects on the grounds that the records are inadmissible character evidence. Should the court sustain the objection? A Yes, because the character of a victim can be established only by reputation or opinion evidence. B Yes, because there is no evidence that the incidents involving the three patrons were based on the same facts as the defendant's claim. C No, because the records were authenticated. D No, because the character trait of a victim may be established by opinion evidence, reputation evidence, or specific acts of misconduct.

A The court should sustain the objection because the records are evidence of specific bad acts. The Federal Rules permit a defendant to introduce evidence of a bad character trait of the alleged victim if it is relevant to the charge or the defense, but limit it to reputation and opinion evidence. Evidence of specific acts of the person in question that demonstrates that person's character is permitted only in a few instances, such as if the acts are relevant to some issue other than disposition to commit the crime charged. Here, no issue is raised by this evidence other than the bouncer's propensity to use excessive force. (A) is therefore correct and (D) is wrong. (B) is wrong because the facts do not have to be identical if evidence of bad acts were otherwise admissible. (C) is wrong; documentary evidence, even if fully authenticated and relevant, may be excluded if it violates a rule of competency, such as the rule for character evidence. Here, the objection should be sustained because the document is improper evidence of a specific bad act.

If a police officer applies for a search warrant and the officer's affidavit of probable cause is based on information obtained from informers, it must include __________. A) enough information to allow the magistrate to make a common sense evaluation of probable cause B) information regarding the basis of the informer's knowledge C) the identity of the informer D) information regarding the informer's reliability and credibility

A The sufficiency of a police officer's affidavit of probable cause is determined by the totality of the circumstances. The affidavit need not contain any particular fact about the informer, as long as it includes enough information to allow the magistrate to make a common sense evaluation of probable cause (i.e., that the information is trustworthy). In obtaining a search warrant, the police may use an informer without revealing his identity, although the identity may have to be revealed at or before trial if the informer is a material witness. Formerly, the affidavit had to include information regarding the reliability and credibility of the informer (e.g., she has given information five times in the past and it has been accurate) and her basis for the knowledge (e.g., she purchased cocaine from the house to be searched). These are still relevant factors, but are no longer prerequisites.

Felony murder generally requires that: A The killing be committed during the course of the felony, the felony must be independent of the killing, and the death must have been a foreseeable result of the felony B The killing be committed during the course of the felony, the death must have been a foreseeable result of the felony, and the defendant must be convicted of the underlying felony C The death must have been a foreseeable result of the felony, the felony must be independent of the killing, and the defendant must have been convicted of the underlying felony D The killing be committed during the course of the felony, the felony must be independent of the killing, and the defendant must be convicted of the underlying felony

A To convict a defendant of felony murder, the prosecution must prove, beyond a reasonable doubt, that the defendant committed a felony (i.e., he is factually guilty of the felony). However, the defendant need not actually be convicted of the underlying felony if the statute of limitations for the felony has expired. The killing must take place while the felony is being committed. When the defendant reaches "a place of temporary safety," the felony is deemed terminated. The felony must be independent of the killing (e.g., the felony of manslaughter cannot be the underlying felony for felony murder). Finally, most states require that the death must be a foreseeable result of the commission of the felony.

For Miranda purposes, the determination of whether a person is in custody is a(n): A Objective test B Subjective test C Mixed objective and subjective test

A Whether a person is in custody is judged objectively—it depends on whether a reasonable person under the circumstances would think that his freedom of action is denied in a significant way. For example, if police officers handcuff a suspect in his own home, he is clearly in custody. Similarly, if they surround a person in the middle of the night in his own bedroom, awaken him, and begin questioning him, custody will be found. It is not true that custody depends on the subjective views of the person undergoing interrogation, or the interrogator. As discussed above, the test is objective—what a reasonable person would think.

DOES THE EXCLUSIONARY RULE APPLY? 1. Rule: 3. Need for Government Action a. For rule to apply, the evidence or information must be obtained as a result of government conduct. b. Categories: i. police ii. informants iii. security guards who have been deputized Ex.: Police employed by subdivision or private college.

A judicially crafted remedy that bars government use, in criminal trials, of information or material secured as a result of a violation of a federal statute or the Defendant's Fourth, Fifth, or Sixth Amendment rights. The main objective is to deter misconduct by law enforcement.

WHEN A SEIZURE OF PROPERTY OCCURS

A seizure of property occurs when the government meaningfully interferes with a person's possessory interest in the property.

Warrantless Arrest:

A warrant generally is not required before arresting a person in a public place, even if the police have plenty of time to obtain one.

HABEAS CORPUS

After a defendant has been convicted and has unsuccessfully appealed her conviction, she may generally still attack her conviction collaterally, usually by applying for a writ of habeas corpus. This proceeding focuses on the lawfulness of a detention, naming the person having custody as the respondent.

If more than _______ months' imprisonment is authorized, the offense is considered "serious" for determining whether a defendant has a constitutional right to a jury trial. A) nine B) six C) three D) twelve

B An offense is considered serious, making a jury trial a constitutional right, when more than six months' imprisonment is authorized.

Miranda warnings __________ need to be given before a suspect is interrogated by a civilian working for the police. A) Always B) May C) Never

B Miranda warnings may need to be given, depending on whether the suspect knows this person is employed by the police. Mirandagenerally applies only to interrogation by the publicly paid police. It does not apply where interrogation is by an informant who the defendant does not know is working for the police. The rationale is that the warnings are intended to offset the coercive nature of police-dominated interrogation, and if the defendant does not know that he is being interrogated by the police, there is no coercive atmosphere to offset.

The Sixth Amendment right to counsel applies __________. A) only at interrogations performed by one known to be a government agent B) only after adversary judicial proceedings have begun C) whenever an informant is placed in the defendant's cell D) whenever a defendant gives a blood sample

B The Sixth Amendment right to counsel applies only after adversary judicial proceedings have begun (e.g., formal charges have been filed). It is not true that the Sixth Amendment applies whenever an informant is placed in the defendant's cell. It applies when an informant is placed in a defendant's cell after adversary judicial proceedings have been initiated. But there is no Sixth Amendment violation just because an informant is placed in the defendant's cell after charges are filed. There is a violation only if the informant does something designed to elicit incriminating remarks. The choice providing that the Sixth Amendment applies only at interrogations performed by one known to be a government agent is too limiting to be correct. A limitation like this one applies to Miranda warnings. The warnings must be given before interrogation by one known to be a government agent in order to offset the coercive nature of police interrogation. A similar rule does not apply under the Sixth Amendment right to counsel. The Sixth Amendment right to counsel applies to all post-charge interrogations, whether or not the defendant knows he is speaking to a government informant. Finally, the blood sample choice is incorrect. The Sixth Amendment right to counsel applies at critical stages of a criminal prosecution after formal proceedings have begun, and the taking of a blood sample is not a critical stage.

While walking on patrol in a commercial district in the early evening, a police officer noticed that a light was on in the back window of a machine shop. Curious about what was going on inside, the officer went to the back of the building and tried to look through the window of the shop, but it had been painted on the inside so that only a strip about three inches at the top, eight feet above ground level, was still transparent. The officer quietly brought two trash cans from a neighboring business over to the window, stood on them and saw, through the strip of unpainted window, that the shop owner's son was inside with a friend, sucking white powder into his nose through a rolled-up tube of paper from off a small mirror. Recognizing that the shop owner's son was snorting cocaine, the officer knocked at the front door to the shop, and the son let him in. The officer immediately arrested the owner's son and his friend. In the back room of the shop through whose window he had peered, the officer found and seized several grams of cocaine, a razor blade, and a mirror. In the subsequent prosecution of the shop owner's son for possession of cocaine, the owner's son seeks to bar introduction of the cocaine, mirror, and razor blade into evidence. Will his motion be granted? A) Yes, because the officer could not have known that the owner's son was snorting cocaine absent a chemical test of the substance being snorted. B) Yes, because the officer violated the owner's son's reasonable expectation of privacy. C) No, because the search was incident to a valid arrest. D) No, because the owner's son consented to the officer's entry into the shop.

B The shop owner's son had a reasonable expectation of privacy, as evidenced by the obscuring of the window so that neighbors could not see into the shop. Hence, the officer's search would have to be based on a valid warrant or qualify under one of the exceptions to the warrant requirement. Putting trash cans next to a back window to climb upon and peering into the window through a narrow opening eight feet above the ground would be considered a violation of the owner's son's Fourth Amendment rights and not a "plain view" of criminal activity. Because the seizure of the cocaine, mirror, and razor was based on the illegal search, the evidence could not be used by the state. (A) is wrong; absolute certainty of illegal activity is not required for a valid search. A reasonable belief is required. (C) is wrong. The arrest itself is probably invalid, and in any event a search of the next room would not be an area within the immediate control of the defendant. (D) is wrong. Consent to enter the shop is not a consent to search the back room.

Miranda warnings are required as a prerequisite to the admissibility of confessions resulting from custodial police interrogation. Which of the following statements is true about the interrogation requirement under Miranda? A) Spontaneous statements violate Miranda if made before Miranda warnings are given. B) The term "interrogation" includes any police tactic that officers should know is likely to elicit an incriminating response. C) Allowing a suspect to talk to his wife in the presence of the police can constitute interrogation. D) Routine booking questions usually will be considered interrogation.

B The term "interrogation" includes any police tactic that officers should know is likely to elicit an incriminating response. It is not limited to direct questioning. The Supreme Court has held that for purposes of Miranda, allowing a suspect to talk to his wife in the presence of the police DOES NOT constitute interrogation. The Supreme Court has also held that routine booking questions do not constitute interrogation (because information about a suspect's name or address usually is not incriminating). Finally, if a spontaneous statement is made before Miranda warnings are given, there is no Miranda violation. Spontaneous statements are statements not made in response to any police conduct. Where the suspect blurts out information upon seeing the police, there is no interrogation.

The following are all exceptions to the Fourth Amendment warrant requirement, allowing for warrantless searches in situations where quick action is needed: A) Evanescent evidence; emergencies that threaten health or safety; investigation of a murder scene B) Hot pursuit of a fleeing felon; evanescent evidence; emergencies that threaten health or safety C) Hot pursuit of a fleeing felon; emergencies that threaten health or safety; investigation of a murder scene D) Hot pursuit of a fleeing felon; evanescent evidence; investigation of a murder scene

B There is no exception to the warrant requirement based on the need to investigate a murder scene. Generally, if the owner of the location where the body was found objects, a warrant must be obtained. There is an exception to the warrant requirement for hot pursuit of a fleeing felon. Police officers in hot pursuit of a fleeing felon may make a warrantless search and seizure. They may even pursue the suspect into private dwellings. There also is an exception to the warrant requirement for evanescent evidence, such as scrapings of tissues from under a suspect's fingernails, which could be washed away. Whether such a warrantless search is reasonable is judged by the totality of the circumstances. If by its nature the evidence is likely to disappear before a warrant may be obtained, the evanescent evidence exception applies. Finally, there is an exception to the warrant requirement when the police are acting in emergency situations threatening immediate health or safety. This includes situations where the police see someone injured or threatened with injury. For example, if a police officer is answering a public disturbance call and sees someone being beaten within the premises when the officer arrives, the officer can go in without a warrant to help the victim of the beating. This is often referred to as the community caretaker exception.

If a defendant is convicted and unlawfully obtained evidence was admitted at trial, on appeal the: A conviction will be overturned if the defendant can demonstrate by a preponderance of the evidence that the admission was harmful. B conviction will be overturned unless the government can show beyond a reasonable doubt that the admission was harmless. C conviction will be overturned if the defendant can demonstrate beyond a reasonable doubt that the admission was harmful. D conviction will be overturned automatically.

B Following a conviction at a trial in which unlawfully obtained evidence was admitted, on appeal, the conviction will be overturned unless the government can show beyond a reasonable doubt that the admission was harmless. A conviction will NOT be overturned automatically because unlawfully obtained evidence was admitted; the harmless error test applies, which means the conviction will stand if the government can prove that the erroneously admitted evidence likely did not have any effect on the conviction. Because the government has the burden of proof, the two choices requiring the defendant to demonstrate that the admission was harmful are incorrect.

Assuming all other elements are met, in which of the following circumstances would the defendant most likely not be convicted of homicide? A) The defendant stabs the victim in the heart; at exactly the same time, another person shoots the victim in the head. Either act would have independently caused the victim's death, but it is unclear which act caused the victim's death. B) The defendant stabs the victim but does not kill him; due to construction at the hospital where the victim is admitted for treatment of the stab wound, the victim contracts asbestosis and dies from it one year later. C) The defendant stabs and kills a victim who has terminal cancer. D) The defendant stabs the victim, who has hemophilia; the victim bleeds to death as a result of the stabbing.

B If the defendant stabs the victim but does not kill him, and due to construction at the hospital where the victim is admitted for treatment of the stab wound, the victim contracts asbestosis and dies as a result one year later, the defendant is not likely to be convicted of homicide. To be convicted of homicide, the defendant must have actually and proximately caused the death of the victim. An intervening act that presents a foreseeable risk will generally not break the chain of causation. However, an unforeseeable risk, such as an injury due to hospital construction, will most likely break the chain of causation. Stabbing and killing someone with terminal cancer is likely to be considered homicide. A defendant may be guilty of killing a victim who was going to die anyhow. Actual and proximate causation is not broken if the defendant ends the victim's life prematurely, even by a short time. If a defendant stabs a victim while another person shoots the victim, the defendant is likely to be found guilty of homicide. Simultaneous acts by multiple persons may be considered independent, sufficient causes of a single result. Thus, multiple persons may be convicted of homicide even though there was only one death. A defendant who stabs a victim with hemophilia who bleeds to death is likely to be found guilty of homicide. Any preexisting conditions that make a person more susceptible to death are essentially disregarded. The defendant "takes the victim as he finds him."

What is the remedy for an unlawful arrest? A The invalid arrest can be used as a defense to the charges asserted B The government generally cannot use at trial evidence that was discovered during the arrest C The police must release the arrestee D There is no remedy if the arrest was based on probable cause

B If the police make an unlawful arrest, generally they cannot use evidence at trial that was discovered during the arrest. This is an application of the exclusionary rule, which prohibits introduction into evidence at trial of items that were unlawfully seized or that are derived from an unlawful search or seizure. The police need not release the arrestee who was unlawfully arrested. If the police have probable cause to detain the arrestee, the fact that the arrest was made in an unlawful manner is not grounds for releasing the defendant. Nor can the invalid arrest be used as a defense to the charges asserted, for reasons similar to those above. Generally, an unlawful arrest itself has no impact on the prosecution. The statement that the arrestee has no remedy if the arrest was based on probable cause is too broad to be true. Even if an unlawful arrest is based on probable cause, the arrestee can move to suppress any evidence derived from the unlawful arrest, as discussed above. The defendant might also be able to bring a "section 1983 action"—a constitutional tort action available when a person is deprived of a constitutional right under color of law. Q

A man and a woman were arrested and charged with a series of armed robberies. Each suspect was given Miranda warnings, and different interrogation teams questioned each suspect separately. Upon being questioned, the man told the police, "I'm not going to talk until I see a lawyer." An officer responded, "You might want to reconsider, because your partner has already confessed, and she's implicated you in the crimes." The man then told the police that he wanted to talk to the woman privately. The police escorted the man to the woman's cell, locked him in with her, and left. Unbeknownst to either of them, the police had bugged the woman's cell and recorded both the man and the woman making self-incriminating statements during their meeting. The man made no further statements to the police on advice of counsel, whom he called immediately after his conversation with the woman. The man was put on trial first, and the prosecution sought to introduce into evidence tapes of the bugged conversation between the man and the woman. The defense made a motion to suppress the evidence. Should the court grant the motion to suppress? A Yes, because the evidence is the fruit of a wiretap that violated the Fourth Amendment. B Yes, because the police created a situation likely to induce the defendant to make an incriminating statement. C No, because there is no expectation of privacy in a jail cell. D No, because the conversation constituted a waiver of the man's Miranda rights.

B The conversation should be suppressed because the police conduct violated the man's Sixth Amendment right to counsel. The Sixth Amendment provides that in all criminal prosecutions a defendant has a right to the assistance of counsel at all critical stages after formal proceedings have begun. For Sixth Amendment purposes, a criminal prosecution begins when adversary judicial proceedings have commenced, such as the filing of formal charges in this case. Because custodial interrogation is a critical stage of prosecution, the Sixth Amendment is violated by post-charge interrogation unless the defendant has waived his right to counsel. Interrogation includes not only direct questioning, but also any other conduct by the police intended to elicit a response. The police conduct here (telling the man that the woman had implicated him and then bugging the conversation) constitutes prohibited interrogation. [See Maine v. Moulton (1985)] (A) is incorrect because the wiretap was not an illegal search under the Fourth Amendment. Wiretapping and other forms of electronic surveillance are subject to the Fourth Amendment prohibition of unreasonable searches and seizures. However, to have a Fourth Amendment right, a person must have a reasonable expectation of privacy with respect to the place searched or the item seized. In a different context, the Supreme Court has held that prisoners have no reasonable expectation of privacy in their cells or in any personal property that they have in their cells. [Hudson v. Palmer (1984)] Hence, neither defendant can assert a Fourth Amendment claim based on the wiretap, because they had no reasonable expectation of privacy in the jail cell. The fact that there was no expectation of privacy does not make choice (C) correct, however. Even though he probably cannot claim that the bugging was an unreasonable search under the Fourth Amendment, the man can claim that it was an interrogation in violation of his Sixth Amendment right to counsel, as discussed above. (D) is incorrect because it is irrelevant. The facts probably would not give rise to a Miranda violation in light of the Court's ruling in Illinois v. Perkins(1990) that Miranda does not apply unless interrogation is by someone known to be a police officer (on the rationale that Miranda is merely a prophylactic rule designed to offset the coercive nature of a custodial interrogation by a police officer). In any case, Mirandarights and Sixth Amendment rights to counsel can only be waived knowingly, and so the man's ignorance of the fact that the cell was bugged precludes a finding of waiver here.

At the defendant's trial for murder, facts were introduced that the defendant acted in the heat of passion. After a lengthy trial, the defendant was convicted of manslaughter. On appeal, the conviction was reversed on procedural grounds. The state immediately moved to retry the defendant, again bringing murder charges against her. The defendant moved to strike the murder charge, and the court refused to grant the motion. After the second trial, the defendant was again convicted of the lesser charge of manslaughter. The defendant appeals the second conviction, claiming that it violated her constitutional rights. May the second conviction stand? A No, because the state could not refile charges after the acquittal. B No, because the state could not retry the defendant for murder under the circumstances. C Yes, because the state could retry the defendant because the manslaughter conviction was overturned. D Yes, because the defendant was reconvicted of the lesser charge again, so any error was harmless.

B The second conviction will not stand. The Double Jeopardy Clause prohibits retrying a defendant whose conviction has been reversed on appeal for any offense more serious than that for which she was convicted at the first trial. This right is violated by retrial for a more serious offense, even if at the second trial the defendant is only convicted of an offense no more serious than that for which she was convicted at the first trial. Thus, (B) is correct because the state could not retry the defendant for murder. (A) is an incorrect statement of the law. The state may retry the defendant, subject to the limitation discussed above. (C) is incorrect. Although a retrial is permissible, the retrial is subject to the limitation discussed above. (D) is also incorrect. Such a violation of the Double Jeopardy Clause is not considered to be harmless even if the defendant is convicted of the same lesser offense again.

What standard must the police meet in order to seize a person for investigatory purposes? A The police may not seize a person for investigatory purposes absent probable cause to arrest B The police may not seize a person for investigatory purposes unless they have at least reasonable suspicion to investigate based on articulable facts C The police may not seize a person for investigatory purposes absent probable cause to investigate D The police may seize a person for investigatory purposes as long as they have at least a scintilla of suspicion

B The statement that the police may not seize a person for investigatory purposes unless they have at least reasonable suspicion to investigate based on articulable facts reflects the correct standard for making an investigatory seizure, also known as a Terry stop. Such stops should be brief and for no longer than necessary to verify or refute the officer's suspicion. Reasonable suspicion requires less than probable cause, but more than a vague notion or gut feeling. The suspicion can be based on personal observation, reliable tips, police flyers, a bulletin, or the like. Whether the standard is met is judged under the totality of the circumstances. The choices indicating that the police may not seize a person for investigatory purposes absent probable cause are both incorrect, because, as indicated above, probable cause is not required. Reasonable suspicion is sufficient. Moreover, the choice requiring probable cause to arrest also is incorrect because a Terry stop can be made for suspicion of criminal activity; investigatory stops are not limited to cases where the police already believe an arrest is warranted. It is not correct that the police may seize a person for investigatory purposes as long as they have at least a scintilla of suspicion, because a scintilla is a very small amount. The scintilla of proof standard is too little suspicion to satisfy the Fourth Amendment requirement that searches and seizures be reasonable. It is sometimes used as the standard for determining whether a case may be taken from a jury and decided on a motion for summary judgment (i.e., if there is a scintilla of evidence regarding a material issue, the case should not be taken from the jury).

The defendant was on trial for murder. The defendant called a witness to testify to an alibi. On cross-examination of the witness, the prosecutor asked, "Weren't you on the jury that acquitted the defendant of another criminal charge?" What is the best reason for sustaining an objection to this question? A The question goes beyond the scope of direct examination. B The probative value of the answer would be substantially outweighed by its tendency to mislead. C The question is a leading question. D Prior jury service in a case involving a party renders the witness incompetent.

B This question raises several different issues: competency of witnesses, use of leading questions on cross-examination, the proper scope of cross-examination, and the probative value/prejudicial impact balancing test. Through a process of elimination, (B) emerges as the correct answer. (D) is incorrect. Under the Federal Rules, virtually all witnesses with personal knowledge are competent to testify. [Fed. R. Evid. 601] A witness is not rendered incompetent simply by having served on a jury in a prior case involving a party to the current suit. Such prior jury service might render the witness's testimony unpersuasive, but it would not make it inadmissible. (C) is incorrect because ordinarily, leading questions are permitted on cross-examination. [Fed. R. Evid. 611(c)] The prosecutor's question is a leading question, but that is perfectly permissible, especially in a case like this, where the alibi witness is not "friendly" toward the prosecution. (A) is incorrect because cross-examination is generally limited in scope to the subject matter of the direct examination and matters affecting the credibility of the witness [Fed. R. Evid. 611(b)], and the prosecutor's question is, in a roundabout way, an attempt to impeach the witness's credibility. The implication behind the question is that if the witness had served on a jury that acquitted the defendant of another criminal charge, the witness would be inclined to think the defendant innocent of the pending charge. Alternatively, the implication behind the question could be that the witness is the kind of person who is "soft on crime" and for that reason is not a credible witness. In either event, because the question is an attempt to impeach the witness's testimony, it is within the proper scope of cross-examination. This leaves (B) as the remaining correct answer. (B) is not unquestionably correct, because the probative value/prejudicial impact balancing test found in Rule 403 is weighted heavily toward admission of evidence. For evidence to be excluded under this balancing test, its probative value must be substantially outweighed by its prejudicial impact. Nevertheless, in this case, a plausible reason for sustaining an objection to the prosecutor's question is that the probative value of the answer would be substantially outweighed by its tendency to mislead. The question and answer would inevitably let the jury know that the defendant had been previously charged with a crime. This information could be highly prejudicial to his defense. Because the question and answer have little probative value (the negative inferences pertaining to the witness's credibility being very weak), it is reasonable to sustain an objection to the question on the basis that its probative value is substantially outweighed by its prejudicial impact.

ACTIVITY A teenager stole a car from the homeowner's driveway after she had left the keys in the car overnight. He was stopped for speeding, and a license plate check indicated that the car had been reported stolen. The teenager was charged with common law larceny. At trial, his defense was that he intended to return the car to the owner's home the following morning before anyone realized it had been taken. The trial judge instructed the jury that while the state must prove the case beyond a reasonable doubt, the defendant has the responsibility to prove his defense by a preponderance of the evidence. The court further instructed the jury that if it found by a preponderance of the evidence that the teenager intended to return the car, it should find him not guilty. The teenager was convicted; he appealed on the ground that the jury instructions were erroneous. Should the teenager's conviction be reversed? A) Yes, because in a criminal case the state must prove all disputed issues beyond a reasonable doubt. B) Yes, because the instruction placed an improper burden of proof on the defendant. C) No, because intent to return is not a defense to the charge. D) No, because the jury instructions were correct.

B. burden should have remained on prosecution A. state has proving every element of offense beyond reasonable doubt, but sometimes defendant puts up defense, it's okay to put burden of proof on defendant by preponderance of evidence C. D.

A man and a woman were arrested and charged with conspiring to blow up a federal government building. After being given Miranda warnings, they were questioned separately and each of them gave a written confession. The confessions interlocked with each other, implicating both of the defendants as being involved in every stage of the conspiracy. Subsequently, the woman attempted to retract her confession, claiming that it was false. At a preliminary hearing, the judge rejected her claim. Both defendants were tried together, and the prosecutor introduced both confessions into evidence. At trial, the woman testified that she was not involved in any conspiracy and that her confession was fabricated. Both defendants were found guilty by the jury. The woman challenges her conviction on appeal because of the admission of the man's confession. If the woman succeeds, what is the likely reason? A) The man's confession was more incriminatory to her than her own confession. B) The jury was not instructed to consider the man's confession as evidence only of his guilt and not of the woman's. C) The man refused to testify at trial and therefore was not subject to cross-examination regarding his confession. D) The man testified at trial and was subject to cross-examination but denied making the confession attributed to him.

C If the woman prevails in her challenge to the admission of the man's confession, it will be because the man could not be cross-examined regarding his confession. Under the Sixth Amendment, a defendant in a criminal prosecution has the right to confront adverse witnesses at trial. If two persons are tried together and one has given a confession that implicates the other, the right of confrontation generally prohibits the use of that statement because the other defendant cannot compel the confessing co-defendant to take the stand for cross-examination. A co-defendant's confession is inadmissible even when it interlocks with the defendant's own confession, which is admitted. If the man refused to take the stand and subject himself to cross-examination, his confession was not properly admitted because it violated the woman's Confrontation Clause rights. (A) is incorrect because the fact that the man's confession incriminates the woman more than her own confession is not relevant. Just the interlocking nature of the man's confession with the woman's confession makes it more damaging by making it harder for the woman to claim that her confession was false. (B) is incorrect because the Supreme Court has held that instructing the jury to consider the confession only as going to the guilt of the confessing defendant is inadequate to avoid Confrontation Clause problems, because the risk that the jury will not follow the limiting instructions is too great in this context. (D) is incorrect. Confessions of a co-defendant may be admitted if (i) all portions referring to the other defendant can be eliminated (so that there is no indication of that defendant's involvement), (ii) the confessing defendant takes the stand and subjects himself to cross-examination regarding the truth or falsity of the statement, or (iii) the confession of the nontestifying co-defendant is being used to rebut the defendant's claim that his confession was obtained coercively, and the jury is instructed as to that purpose. Even if the co-defendant denies ever having made the confession, as stated in choice (D), the opportunity at trial to cross-examine the co-defendant satisfies the Confrontation Clause.

A defendant was convicted after a jury trial of violation of federal statutes prohibiting the sale of automatic weapons to foreign nationals. It was established at trial that the defendant had purchased a number of stolen United States Army heavy machine guns and attempted to ship them abroad. The trial court expressly based its imposition of the maximum possible sentence for the conviction on the defendant's refusal to reveal the names of the persons from whom he purchased the stolen weapons. His counsel argues that this consideration is reversible error. If the defendant appeals the sentence imposed, what should the appeals court do? A) Reverse the trial court, because the consideration of the defendant's silence violates his Fifth Amendment privilege against self-incrimination. B) Reverse the trial court, because the consideration of collateral circumstances in sentencing violates his due process rights. C) Affirm the trial court, because the right to remain silent granted by the Fifth Amendment does not include the right to protect others from incrimination. D) Affirm the trial court, because citizens must report violations of the criminal statutes.

C The appeals court should affirm the trial court because the right to remain silent does not include the right to protect others from incrimination. The defendant was not privileged to refuse revealing the names of the stolen weapon sellers. The United States Supreme Court held, in Roberts v. United States (1980), that a defendant's refusal to cooperate with an investigation of the criminal conspiracy of which he was a member may properly be considered in imposing sentence. This is because the Fifth Amendment right to remain silent does not afford a privilege to refuse to incriminate others. (C) is therefore correct and (A) is incorrect. (B) is incorrect because the court's consideration of the defendant's refusal to cooperate does not violate due process. (D) is not an accurate statement of the law.

The statutes of a state define the following crimes (with the most serious listed first): First degree murder—Premeditated or intentional killing. Felony murder—Killing while in the act of committing a common law felony. Second degree murder—Killing with reckless disregard for the safety of others. Manslaughter—Killing with adequate provocation or through criminal negligence. A competitive camp counselor who was determined to have her team win a relay race decided to put a colorless and odorless drug into the other teams' water bottles. The counselor wanted the other teams to become sick to their stomachs so that they could not run as fast in the race. The counselor knew that people could become very ill, or even die, if they consumed too large a quantity of the drug but she only intended to place a small amount in each water bottle. The day before the race, the counselor went into the nurse's office and took a bottle of the drug. The next day she woke up early and went to the cafeteria to put small quantities of the drug into the other teams' water bottles. She unintentionally put a large amount in a few of the water bottles. Several campers became extremely ill and one eventually died. What is the most serious crime for which the counselor may be convicted? A) First degree murder. B) Felony murder. C) Second degree murder. D) Manslaughter.

C The counselor may be convicted of second degree murder. Under the statute provided in the question, second degree murder is a killing committed with a reckless disregard for the safety of others. Here, the counselor consciously disregarded a substantial and unjustifiable risk that the drug she was placing in the water bottles could be seriously harmful or even fatal. Thus, the counselor may be convicted of second degree murder. (A) is wrong. The counselor could not be convicted of first degree murder because she lacked the necessary intent to kill; the counselor's only intent was to make the other team's members sick. (B) is wrong. The counselor cannot be convicted of felony murder because she did not commit a felony; a majority of jurisdictions would consider simple battery to be a misdemeanor. Furthermore, the felony generally must be independent of the killing. A battery (or aggravated battery) would not be considered independent of the conduct which kills (although it might be considered to be a valid basis for a manslaughter conviction). (D) is not as good an answer as (C). Although a jury could find that her actions were criminally negligent, resulting in manslaughter, a jury could also find that she acted with reckless disregard, resulting in second degree murder. Thus, given that the call of the question asks for the most serious crime for which the counselor could be convicted, (C) is correct.

If the police have probable cause to search an automobile, they __________ search the __________ belongings as well. A May not; driver's or passenger's B May; driver's, but not passenger's C May; driver's or passenger's

C As an exception to the search warrant requirement, if the police have probable cause to search an automobile for contraband, they may search the passenger's belongings as well those of the driver. They may search anywhere in the automobile in which items for which they have probable cause to search may be hidden. The rationale for this rule is that a passenger, like the driver, has a reduced expectation of privacy in an automobile. If the police have probable cause to believe that a vehicle contains contraband or fruits, instrumentalities, or evidence of a crime, they do not need to obtain a search warrant. The rationale for this exception is that automobiles are mobile, so they will not likely be available for a search by the time an officer returns with a warrant.

In which of the following does one have a reasonable expectation of privacy for Fourth Amendment purposes? A The sound of one's voice B The record of one's bank account C The feel of one's luggage D The smell of one's luggage

C The Supreme Court has held that one does have a reasonable expectation of privacy in the feel of one's luggage. While one usually does not have a privacy interest in things held out to the public, the Court found that people generally do not hold their luggage out to the public to be squeezed. Thus, squeezing luggage to discern its contents constitutes a search. Interestingly, though, the Supreme Court has held that one does NOT have a privacy interest in the smell of one's luggage—the smell is held out to the public—so drug and bomb sniffing dogs can sniff away and this does not constitute a search. In addition to not having a privacy interest in the smell of one's luggage, the Supreme Court has held a person does not have a reasonable expectation of privacy in the following things held out to the public: (i) The sound of one's voice; (ii) Account records held by a bank; (iii) One's handwriting; (iv) The paint on the outside of one's car; and (v) Magazines one offers for sale.

Assuming all other elements of homicide are met, in which of the following circumstances would the defendant most likely not be convicted of homicide? A The defendant stabs the victim, who has hemophilia; the victim bleeds to death as a result of the stabbing B The defendant and another person shoot the victim at exactly the same time; either act would have independently caused the victim's death, but it is unclear which act caused the victim's death C The defendant stabs the victim and on the way to the hospital, a tree is hit by lightning and a limb falls on the ambulance crushing the back portion and killing the victim D The defendant kills a victim who had been poisoned by a fatal poison just minutes before

C The best scenario for the defendant not to be convicted of homicide is where the defendant stabs the victim and on the way to hospital, the tree branch crushes the ambulance. To be convicted of homicide, the defendant must have actually and proximately caused the death of the victim. An intervening act that presents a foreseeable risk will generally not break the chain of causation. However, an unforeseeable risk, such as an act of nature, will most likely break the chain of causation. Killing someone who has been terminally poisoned is likely to be considered homicide. A defendant may be guilty of killing a victim who was going to die anyhow. Actual and proximate causation is not broken if the defendant ends the victim's life prematurely, even by a short time. If a defendant and another person both shoot the victim, the defendant is likely to be found guilty of homicide. Simultaneous acts by multiple persons may be considered independent, sufficient causes of a single result. Thus, multiple persons may be convicted of homicide even though there was only one death. A defendant who stabs a victim with hemophilia who bleeds to death is likely to be found guilty of homicide. Any preexisting conditions that make a person more susceptible to death are essentially disregarded. The defendant "takes the victim as he finds him."

If a defendant is convicted at a trial in which unlawfully obtained evidence was admitted, on appeal, the conviction will be: A Upheld unless the defendant can prove by a preponderance of the evidence that the error was not harmless B Upheld if the government can prove by a preponderance of the evidence that the error was harmless C Upheld only if the government can prove beyond a reasonable doubt that the error was harmless D Upheld unless the defendant can prove beyond a reasonable doubt that the error was not harmless

C The conviction will be upheld if the government can prove beyond a reasonable doubt that the error was harmless. A conviction will not necessarily be overturned merely because improperly obtained evidence was admitted at trial; the harmless error test applies, and a conviction can be upheld if the conviction would have resulted despite the improper evidence. In other words, the conviction will stand if the government can prove that the erroneously admitted evidence likely did not have any effect on the conviction. A preponderance of the evidence is a more lenient standard than the required reasonable doubt standard. The choices placing a burden on the defendant both are incorrect; the burden is on the government.

The police received a tip from a reliable informant that a former student at the local university was selling narcotics. A brief investigation revealed that the former student, a college dropout, still hung around the university campus, had no visible means of support, and yet drove a large luxury car and wore flashy clothing and jewelry. The police picked up the former student the next time he showed up on campus, took him to the station, and questioned him all night long without a break and without letting him communicate with anyone else. When the former student tired from the interrogation, he admitted that he sold cocaine to his friend, who is a current student at the university. Based on this information, the police went to the current student's dormitory room. When they arrived, they found the door open but no one was in the room. The police entered, searched the room, and discovered a vial of white powder. Later laboratory tests established the powder to be cocaine. The former student was then charged with the sale of narcotics. At his trial, the prosecution attempted to admit the cocaine discovered in the dormitory room into evidence. What is the former student's best argument for preventing the cocaine from being admitted into evidence? A The search of the dormitory room was conducted without a warrant and without consent. B The police arrested the former student without a warrant. C The former student's confession was not voluntary under the circumstances. D The police failed to give the former student Miranda warnings.

C The former student's best argument for preventing the cocaine from being admitted into evidence is that his confession was not voluntary. This question is difficult because each of the choices appears to present a good argument for the former student. With regard to (A), the search of the current student's dorm room appears to be an unreasonable search under the Fourth Amendment. However, a person's Fourth Amendment rights against unreasonable search and seizure may be enforced by the exclusion of evidence only at the instance of someone whose own protection was infringed by the search and seizure. Here, the former student cannot assert a possessory interest or reasonable expectation of privacy in the current student's dorm room. Thus, the former student cannot successfully exclude the cocaine on the ground that it was seized in violation of the Fourth Amendment. (B) is incorrect because arrest warrants are usually required only for arrests made in the person's home. Police generally do not need to obtain a warrant before arresting a person in a public place, even if they have time to get a warrant, as long as the arrest is based on probable cause. Here the police had probable cause to arrest the former student, and because he was arrested on the grounds of the campus, the failure of the police to obtain an arrest warrant will be of no help to him. Choices (C) and (D) both focus on improper conduct during the former student's interrogation, but (C) is better because the former student will have a better chance of invoking the exclusionary rule if the confession is involuntary. For confessions to be admissible, the Due Process Clause of the Fourteenth Amendment requires that they be voluntary. While voluntariness is a fact question that is assessed by looking at the totality of the circumstances, the duration and manner of the police interrogation here indicate that the confession probably was the result of actual coercion. If the confession is found to be involuntary, the former student can invoke the exclusionary rule to exclude the cocaine as "fruit of the poisonous tree." In contrast to an involuntary confession, a confession obtained without Miranda warnings, as long as the failure to warn was not purposeful, may not be sufficient to justify excluding the nontestimonial "fruits" of the confession. [See United States v. Patane (2004)] Thus, the involuntariness of the confession, rather than the absence of Miranda warnings, is the best argument for excluding the cocaine.

In most states, in order to convict a defendant of felony murder, the prosecution must, at trial: A Charge and convict the defendant of the predicate felony B Prove that the defendant committed the predicate felony by a preponderance of the evidence C Prove that defendant committed the predicate felony beyond a reasonable doubt D Charge the defendant with the predicate felony

C The prosecution must prove, beyond a reasonable doubt, that the defendant committed the felony on which felony murder liability is based. Generally, there is no need to charge or convict the defendant of the felony. Thus, for example, if the statute of limitations has run on the predicate felony, it will serve as no defense to a felony murder charge, so long as the prosecution can prove, beyond a reasonable doubt, that the defendant committed the predicate felony. (Murder generally does not have a statute of limitations.) Finally, the prosecution must prove each element of a crime by a "beyond a reasonable doubt" standard, not by a preponderance of the evidence, including the commission of a felony for felony murder.

Which of the following generally is not required to convict a defendant of felony murder? A The felony must be independent of the killing. B The killing must be committed during the course of the felony. C The defendant must be convicted of the underlying felony. D Death must have been a foreseeable result of the commission of the felony.

C To convict a defendant of felony murder, the prosecution must prove, beyond a reasonable doubt, that the defendant committed a felony (i.e., he is factually guilty of the felony). However, the defendant need not actually be convicted of the underlying felony if the statute of limitations for the felony has expired. The killing must take place while the felony is being committed. When the defendant reaches "a place of temporary safety," the felony is deemed terminated. The felony must be independent of the killing (e.g., the felony of manslaughter cannot be the underlying felony for felony murder). Finally, most states require that the death must be a foreseeable result of the commission of the felony.

ACTIVITY A woman was arrested, given Miranda warnings, and questioned about an armed robbery. After she asked to speak with an attorney, the police stopped questioning her about the robbery. Several hours later, the police gave the woman a fresh set of Miranda warnings and began to question her about a different robbery. She did not repeat her request for an attorney and instead made several incriminating statements about the robbery. At the woman's trial for the robbery for which she made incriminating statements, the pros- ecution seeks to have her statements introduced into evidence. If the woman's attorney objects on appropriate grounds, how should the court rule? A) Overrule the objection, because the police did not badger the woman into confessing. B) Overrule the objection, because the woman did not renew her request for an attorney after receiving fresh Miranda warnings. C) Sustain the objection, because the police did not honor the woman's request. D) Sustain the objection, because a confession obtained in violation of a defendant's Miranda rights but otherwise voluntary may be used against the defendant.

C Correct!! A. badger b/c not honor right to remain silent - problem is didn't invoke right to remain silent, invoked right to counsel = can't come back B. cannot come back b/c right to counsel invoked D. seems nonsensical

ACTIVITY A city ordinance makes it a misdemeanor of disorderly conduct to smoke on a city bus. A defendant boarded a city bus and lit a cigarette. The bus driver asked the defendant to put out her cigarette. The defendant ignored the driver and took a seat. The driver flagged down a passing police officer and told him that the defendant was smoking on the bus. Meanwhile, the defendant quickly extinguished the cigarette and put the butt in her purse before the officer had boarded the bus. The officer told the defendant that she was under arrest for disorderly conduct. Was the defendant lawfully arrested? A) No, because the defendant had extinguished the cigarette. B) No, because the officer did not have reasonable suspicion. C) Yes, because the officer had probable cause to arrest the defendant. D) Yes, because the informer was a government agent.

C) A) just because someone stopped violating a law doesn't mean they hadn't already violated it - he has bus driver witness B) officer needs probable cause for an arrest D) government agent? doesn't matter in terms of witnesses except in terms of credibility assessment

A defendant's car was stopped by an officer for a minor traffic violation. The officer recognized the defendant as a suspect in a multimillion-dollar bank fraud scheme that had just been discovered by the authorities. She placed the defendant under arrest and gave him Miranda warnings. She then asked for permission to search the trunk of the car. The defendant nodded and unlocked the trunk. The officer searched the trunk and discovered a bag containing what appeared to be cocaine in a compartment in the trunk. When later tests determined that it was cocaine, the authorities added a charge of transporting illegal narcotics to the defendant's indictment. At a preliminary hearing, the defendant moved to have evidence of the cocaine excluded as the result of a search in violation of the Fourth Amendment. Should the court grant the defendant's motion? A) Yes, because one taken into custody cannot give valid consent to a search that would otherwise require a warrant. B) Yes, because the search exceeded the scope of a permissible search incident to a lawful arrest. C) No, because it appears that the defendant's consent was voluntary under the circumstances. D) No, because persons have a lesser expectation of privacy in their vehicles for purposes of the Fourth Amendment.

C) no defendant consented to search of trunk. A) something to this, but also doesn't look great, states a per se rule and there is no per se rule like that B) reasonable officer believed and defendant consented, so not outside scope of consent D) that's true, but doesn't allow suspicious searches

EXAMPLE Defendant suffers appendicitis during trial and jury is dismissed. Defendant can be retried for the same offense when released from the hospital.

manifest necessity - have to be able to retry the defendant

Custody:

Custody occurs when Defendant's freedom of movement is limited to an extent that creates the inherently coercive pressures that arise during a full stationhouse arrest. It is an objective test, based on police behaviors.

Acting on an anonymous telephone call, police went to the defendant's apartment, knocked on the door, and demanded to search it for narcotics. When the defendant refused, the police forced the door open and placed him under arrest. As they were removing him from the apartment, the defendant offered to give the officers "valuable information" in exchange for his release. Before he could say anything else, the defendant was given Miranda warnings by the police. Thereafter, he told the police that he had stored some heroin in his friend's apartment and that he and his friend had been going to sell it. The heroin was recovered, and the defendant was prosecuted for conspiracy to sell narcotics and for possession of narcotics. At his trial, the defendant moved to suppress his statements. Which of the following is the defendant's best argument in support of the motion to suppress? A) The defendant is entitled to know the identity of his accuser, and the state cannot supply this information. B) The police should have given the defendant Miranda warnings prior to entry into the apartment, and the warnings were ineffectual once the defendant offered to give the police information. C) The defendant was intimidated by the forced entry into the apartment, and because the statements were involuntary and coerced, their use against him would violate due process of law. D) The statements were fruits of an unlawful arrest, and though the Miranda warnings may have been sufficient to protect his right against self-incrimination, they were not sufficient to purge the taint of the illegal arrest.

D The entry into the defendant's apartment and his arrest, without a warrant, probable cause, or circumstances permitting an exception from these requirements, were unconstitutional. The statements he made thereafter were fruits of the original unconstitutional arrest and must be suppressed unless the taint was purged. The giving of Miranda warnings was not sufficient. Hence, (D) is the best answer. If probable cause for a warrant is based on information from an informer, usually that informer's identity need not be revealed. Thus, (A) is incorrect. (B) is a misstatement of law. There was no interrogation by the police to trigger the Miranda requirements. (C) is attractive but not as accurate an answer as (D). If the police had been acting with probable cause to arrest, their forced entry into the apartment would not have made the defendant's statements involuntary.

A motorist drove home from work late one night, and fell asleep behind the wheel of his car. His car drifted across the middle of the road and struck another car. The other driver was killed instantly in the collision. Angered by the noise of the collision, a homeowner fired a gun out the window of his house at the car. The bullet struck and killed a bystander. Both the motorist and the homeowner were arrested and charged with common law murder. Which of the defendants likely would be found guilty? A) Both the motorist and the homeowner. B) The motorist. C) Neither the motorist nor the homeowner. D) The homeowner.

D The homeowner likely would be found guilty. At common law, murder was the unlawful killing of a human being with malice aforethought. Malice aforethought could be established with any one of the following states of mind: intent to kill; intent to cause serious bodily harm; the depraved heart killing (a reckless indifference to an unjustifiably high risk to human life); or the commission of a felony. The homeowner would be guilty of murder. Firing a gun out of his window at a car would demonstrate a reckless indifference to a high risk to human life. Thus, (B) and (C) are incorrect. It is unlikely that the motorist would be guilty of murder. While his action might be classified as negligent or even reckless, it would not represent a depraved heart (reckless indifference to life) state of mind. Thus, (A) and (B) are incorrect.

There is an exception to the warrant requirement for searches incident to arrest. While the exception is based in part on the safety of police officers, the arresting officer need not fear for his safety before conducting the search. In certain circumstances, the search may extend beyond the arrestee's person. In which of the following circumstances is the search after arrest not valid without a search warrant or other facts justifying the search? A A police officer searches areas within the arrestee's reach at the time of arrest and discovers a bag of illegal drugs in a nearby kitchen cabinet. B A police officer followed one of two armed bank robbers to the robber's home, arrested him, performed a protective sweep of the entire home to ensure the other robber was not there, and found evidence of the robbery in plain view in a closet. C A police officer arrests a person on domestic battery charges just outside his home, allows the person to reenter to get a pack of cigarettes, accompanies him, searches the drawer in which the person reached for the cigarettes, and finds a vial of crack cocaine. D A police officer searches the interior of the arrestee's car after he was stopped for driving without a license and placed within a squad car.

D A search of the interior of an automobile incident to arrest may be performed only if the arrestee is not secured or if the police have reasonable suspicion that the automobile harbors evidence of the crime for which the arrest was made. Under the facts here, neither of the qualifying circumstances is present. Therefore, the warrantless search exception for searches incident to arrest does not apply and the search was invalid under the Fourth Amendment. The search within the arrestee's reach at the time of arrest (also known as the arrestee's wingspan or grab area) is valid. A search incident to arrest can extend to areas within the arrestee's reach. Moreover, even though the exception is for the protection of police officers, the officer need not have reason to believe that weapons, evidence, or contraband will be found in the area searched, as long as it is within the arrestee's immediate reach. The search of the drawer also is valid. This is an application of the above rule that officers may search not only the arrestee's person, but also his wingspan. An arrestee's wingspan moves as the arrestee moves. The protective sweep within the armed bank robber's home also is valid. A search incident to arrest can include a protective sweep of the premises in which the person was arrested if the arresting officer has reason to believe that accomplices may be present. Here, the arresting officer knew that two armed people had just robbed a bank. He could reasonably fear that the other robber was in the home to which he followed the arrestee.

If a police officer kills an innocent bystander while attempting to stop an enumerated felony in progress, the felon may be criminally liable for felony murder: A if the jurisdiction follows the Redline approach to felony murder B always C if the jurisdiction follows the "agency" approach to felony murder D if the jurisdiction follows the "proximate cause" approach to felony murder

D If a police officer kills an innocent bystander while attempting to stop an enumerated felony in progress, the felon may be criminally liable for felony murder if the jurisdiction follows the "proximate cause" approach to felony murder. In states following the "proximate cause" approach to felony murder, felons are liable for the deaths of innocent victims caused by someone other than a co-felon. The felon will be guilty of felony murder when he puts into operation a series of events that proximately cause the death of an innocent party. Under the Redline approach, felony murder liability does not attach when the person killed is a co-felon. The Redline approach does not cover the situation where an innocent person is the victim. Under the "agency" approach, the person doing the killing must be the defendant or the defendant's agent. In other words, if someone who does not have a stake in the felony (e.g., a police officer, a resisting victim) accidentally kills another, the person committing the felony cannot be held liable for felony murder.

The defendant and an accomplice were on trial together for burglary. Both had given confessions implicating themselves and their accomplice. At trial, the defendant maintained that his confession had been obtained through improper coercion by the police. For the purpose of countering the claim of coercion, the prosecution seeks to place the accomplice's confession into evidence. After objection by the defendant's counsel, the judge agrees to issue a limiting instruction to the jury that the confession is to be considered only with regard to the question of whether the defendant's confession was coerced. May the accomplice's confession be admitted under that condition? A No, because admission of the confession violates the defendant's right of confrontation. B No, unless the accomplice takes the stand and subjects himself to cross-examination regarding the confession. C Yes, as long as all portions of the confession referring to the defendant can be eliminated. D Yes, because the judge's instruction limits consideration of the confession only to the issue of coercion.

D The confession is admissible with the judge's limiting instruction. Where two persons are tried together and one has given a confession implicating the other, the general rule is that the Sixth Amendment right to confront adverse witnesses prohibits the use of such a statement. This problem arises because of the inability of the nonconfessing defendant to compel the confessing co-defendant to take the stand for cross-examination at their joint trial. As exceptions to the general rule, the statement may be admitted if: (i) all portions of the statement referring to the other defendant can be eliminated (so that there is no indication of that defendant's involvement); (ii) the confessing defendant takes the stand and subjects himself to cross-examination with respect to the truth or falsity of what the statement asserts; or (iii) the confession of the nontestifying co-defendant is being used to rebut the defendant's claim that his confession was obtained coercively, in which case the jury must be instructed as to the purpose of the admission. The accomplice's confession, which the prosecution seeks to introduce into evidence, implicates the defendant in the commission of the crimes charged. Consequently, introduction of this confession raises a problem based on the right of confrontation. However, given that the judge will issue the limiting instruction, the confession is admissible. (D) is therefore correct and (A) is incorrect. (B) and (C) are incorrect because neither of those conditions is necessary for the confession to be admitted as long as the judge issues a limiting instruction, as discussed above.

In a criminal battery case brought against the defendant, the prosecutor asked the court to take judicial notice of the fact that a car driven from Chicago to Detroit has to cross state lines. The defense attorney raised no objection, and the judge declared that she was taking judicial notice of the fact as requested by the prosecution. What is the effect of such judicial notice? A To raise an irrebuttable presumption. B To satisfy the prosecutor's burden of persuasion on that issue. C To shift the burden of persuasion on that issue to the defendant. D That the judge should instruct the jury that it may, but is not required to, accept the noticed fact as conclusively proven.

D The effect of the judge's noticing that a car driven from Chicago to Detroit must cross state lines is that the judge will now instruct the jury that it may, but is not required to, accept that fact as conclusively proven. Under the Federal Rules, in a civil case, the court must instruct the jury to accept the judicially noticed fact as conclusive. [Fed. R. Evid. 201(f)] Because this question deals with a prosecution for criminal battery, the applicable rule is that the jury be instructed that the fact that has been judicially noticed may be accepted by it as conclusive, but that the jury is not required to do so. (A) would be correct if this were a civil case. In such an instance, the jury would be instructed to accept as conclusive the judicially noticed fact. This would have the effect of raising an irrebuttable presumption. (B) is incorrect because, in a criminal case, the prosecution has the burden of proving every element of the crime beyond a reasonable doubt. Only the jury can decide, after all of the evidence is in, whether the burden of persuasion is satisfied. (C) is incorrect because the burden of persuasion does not shift from party to party during the course of the trial. The burden of persuasion is never on a criminal defendant.

A defendant charged with driving while intoxicated pleaded not guilty and insisted on a trial. Right before the trial began, he fired his attorney and decided to defend himself. At one point during opening arguments, the defendant began to act like a cat, meowing and chasing an imaginary squirrel out of the courtroom. If no one else raises the issue of the defendant's competency to stand trial, what is the responsibility of the trial judge here? A The trial judge has no responsibility, because the defendant decided to defend himself. B The trial judge has no responsibility, because she cannot decide whether the defendant is competent to stand trial. C The trial judge must raise the issue of competency, because the defendant is representing himself. D The trial judge must raise the issue of competency, because the Constitution obligates her to do so.

D The judge must raise the issue of competency. If it appears to the judge that the defendant might be incompetent, the judge has a constitutional obligation to conduct further inquiry and determine whether in fact the defendant is incompetent. If the defendant is tried and convicted but it later appears that he was incompetent to stand trial, the judge's failure to raise the issue or to request a determination of competency does not constitute a waiver of the competency issue. [Pate v. Robinson (1966)] Therefore, if the trial judge observes the defendant acting in such a way that may indicate he is incompetent to stand trial (e.g., meowing, chasing imaginary squirrels), she should conduct further inquiry to determine the competency of the defendant. (A) is incorrect. As explained above, the trial judge has a duty to conduct an independent inquiry if there is evidence that the defendant may not be competent to stand trial. This would be true even if the defendant were represented by counsel. (B) is incorrect. As explained above, the judge can and should decide the issue of competency if there is evidence that the defendant might not be competent to stand trial. (C) is incorrect. If there is evidence that the defendant might not be competent to stand trial, the judge should conduct an independent inquiry into competency regardless of whether the defendant is representing himself or is represented by an attorney.

A woman was the subject of a murder investigation. The investigation continued for more than two years, with the woman frequently being called in for questioning. Finally, the woman was indicted for the murder. The woman's lawyer filed a motion to dismiss all charges against her, arguing that the excessively long investigatory period violated the woman's constitutional right to a speedy trial. Despite the pending motion, the woman decided that she wanted to "get it over with," and she told the judge that she wished to plead guilty. The judge then explained the charges to the woman and asked her if she understood them. She replied, "Yes." The judge then asked the woman if she understood that she was not required to plead guilty. She responded in the affirmative. Finally, the judge described the maximum sentence and asked the woman if she understood that she could receive the maximum sentence, which was life imprisonment. She again responded, "Yes," and maintained that she still wished to plead guilty. The judge accepted the woman's plea and sentenced her to 30 years' imprisonment in the state penitentiary. Six months later, the woman filed a motion to set aside the guilty plea. Which of the following provides the best argument that the woman has a constitutional basis for relief? A The judge did not rule on the pending motion to dismiss before accepting her guilty plea. B The judge did not attempt to determine if the woman had actually committed the murder. C The judge did not determine whether the files in the prosecutor's office contained any undisclosed exculpatory evidence. D The judge did not determine whether the woman understood that she had a right to a trial by jury.

D The judge's failure to determine whether the woman understood her right to trial by jury indicates that her guilty plea does not satisfy the constitutional requirement that it be "voluntary and intelligent." A guilty plea is a waiver of the Sixth Amendment right to a jury trial. To be a valid waiver, the judge must determine on the record that the guilty plea represents a voluntary and intelligent choice among the alternative courses of action open to the defendant. To ensure that this is the case, the judge should make sure that the defendant is informed of the nature of the charge to which the plea is offered, of the maximum possible penalty, that she has a right not to plead guilty, and that by pleading guilty she waives her right to a trial. If the judge did not determine whether the woman understood that she had a right to a trial by jury, her plea will not be a sufficiently intelligent choice to satisfy the constitutional standard, and therefore will not be immune from a post-sentence attack on it. (A) is incorrect because the woman had no legitimate grounds for her motion to dismiss for violation of her right to a speedy trial. The Sixth Amendment right to a speedy trial does not attach until the defendant has been arrested or charged. Pre-arrest delays do not violate this standard, nor do they violate general due process requirements unless they were in bad faith and prejudice the defendant. Otherwise, the only limitation on pre-arrest delay would be the statute of limitations for the particular crime. Thus, the failure of the judge to rule on the motion to dismiss would not be a good argument for setting aside the woman's guilty plea. (B) is incorrect because most jurisdictions do not require that the record contain evidence of the defendant's guilt or other factual basis for the plea. Unless the defendant claims her innocence while offering a guilty plea, the judge need not determine whether there is evidence to indicate that the defendant actually committed the crime. (C) is incorrect. While the prosecutor has a duty to disclose exculpatory evidence to the defendant, the judge may accept a guilty plea without determining whether the prosecutor has satisfied that duty.

ACTIVITY Dwayne Dealer is sitting on a bench next to Betty Bystander and notices the police approaching. Before the police see him, he stashes a baggie of cocaine in Betty's purse. The police come up to Dwayne and Betty, grab the purse, open it up, and find the cocaine. Can Dwayne successfully challenge the search of Betty's purse? A) Yes, because his cocaine was found in the purse. B) Yes, because the police physically intruded on a constitutionally protected area. C) No, because the purse was located in a public place. D) No, because he had no reasonable expectation of privacy in the purse.

D - best answer, searching purse of Betty B A - not correct because cannot challenge if you have reasonable expectation of privacy in place searched B - that's correct, but it's not his constitutionally protected area C - purse not public place illegal search of Betty's purse

ACTIVITY Police investigating a homicide had probable cause to believe that the defendant had committed it. They then learned from a reliable informant that, a short while ago, the defendant had gone to a friend's house to obtain a false driver's license from the friend, a convicted forger. Believing that the defendant might still be there, the police, without obtaining a warrant, went to the friend's house. They entered the house and found the defendant hiding in the basement. He was arrested and given his Miranda warnings. At the police station, he confessed to the homicide. At a preliminary hearing, the defendant's attorney contends that the confession should be suppressed on Fourth Amendment grounds. Is the court likely to agree? A) Yes, because the police did not have a search warrant to enter the friend's house and there were no exigent circumstances. B) Yes, because the police did not have an arrest warrant for the defendant and there were no exigent circumstances. C) No, because a reliable informant told police that the defendant was in the friend's house. D) No, because the police had probable cause to arrest the defendant.

D) problem presents attenuation doctrine - once take him out of friend's home, continued custody becomes lawful once they leave defendant's home, under attenuation doctrine can interrogate him A) private place (home of 3rd party normally need a search warrant) B) no need an arrest warrant in 3rd party's home C) not enough for search warrant

EXAMPLE Two officers who are transporting Defendant have conversation about a murder Defendant is suspected of having committed, within the earshot of Defendant. This is not considered interrogation unless officers have awareness of Defendant's special susceptibility to such an appeal to conscience.

Defendant suspected of having murdered cab driver, arrested him, driving to station house, gee wouldn't it be a shame if some poor handicapped girl found the gun and killed herself with it; so guy says, ok I will show you gun, I don't anyone to get hurt. Supreme Court said no, not an interrogation, not functional equivalent because they did not know he had a soft spot for disabled children.

AUTOMOBILE STOPS Pretextual Stops:

If police have probable cause or reasonable suspicion to justify the auto stop, they can do so even if they are actually motivated by a desire to investigate some other offense (as to which they lack any reasonable suspicion or probable cause).

AUTOMOBILE EXCEPTION EXCEPTION:

EXCEPTION: Police may not search the contents of a defendant's cell phone under the automobile exception. Because smart phones contain such a large quantity of personal information, police must get a warrant before searching them (unless the search is justified by exigent circumstances). *Justification for exception to warrant requirement: Automobiles are mobile and they are highly regulated.

"Harmless error test."

Even where a court erroneously fails to exclude tainted evidence at trial, the conviction can be upheld if the prosecution can show beyond a reasonable doubt that the conviction would have occurred even without the tainted evidence. Evidence lawfully obtained is overwhelming so conviction would have stood up, court will not overturn.

DOES THE EXCLUSIONARY RULE APPLY? 2. Scope:

If rule applies, illegally obtained evidence and all evidence or information obtained or derived from exploitation of the evidence is barred ("fruit of the poisonous tree" doctrine).

Warrant Search Rule

General Rule: A search is unreasonable unless it is performed in compliance with a valid warrant. a. Was Warrant Approved by Neutral and Detached Magistrate? i. Examples of Test Not Being Satisfied: *magistrate has financial stake in issuing warrant *magistrate does not read the warrant *magistrate takes active role in investigation (police wanted to do search of pornography shop, judge wanted to come down) b. Did Warrant Particularly Describe the Place to be Searched and the Item to be Seized?

ARRESTS AND OTHER DETENTIONS 4th Amendment Rule:

Individuals have a right to be free of "unreasonable...seizures." This rule covers both seizures of property and seizures of the person.

Voluntary Manslaughter IKHPWDAAP

Intentional killing in the heat of passion where defendant acts with adequate provocation

Interrogation:

Interrogation is express questioning or words or actions that the police know or should know were likely to elicit an incriminating response.

A felon planned to break into the rental storage unit next to his that contained valuable electronic equipment. He went to a hardware store to purchase a crowbar. The proprietor sold him the crowbar even though he told her thathe needed it to break into someone's storage unit. After the purchase, the felon went to the storage facility with his friend. The felon told the friend that he had lost the key to his storage unit and did not have time to contact the facil- ity's manager, so they needed to break into the unit to get his equipment. Because the felon had a bad back, the friend pried open the door with the crowbar and carried the equipment out to the car. A silent alarm was triggered and the pair were apprehended shortly after leaving the facility. Can the proprietor and the friend be convicted as accomplices to larceny? (A) Yes as to the proprietor, because she knew that the felon was going to break into someone's storage unit with the crowbar. (B) Yes as to the friend, because he carried out the theft of the items from the storage unit. (C) Yes as to both the proprietor and the friend. (D) No as to both the proprietor and the friend.

Neither the proprietor nor the friend would be convicted as accomplices to larceny. An accom- plice is one who (i) with the intent to assist the principal and the intent that the principal commit the substantive offense (ii) actually aids, counsels, or encourages the principal before or during the commission of the crime. Here, the friend is not liable as an accomplice because nothing in the facts suggests that he disbelieved the felon's claim that it was his own storage unit. Hence, he had no intent to commit larceny, which requires an intent to permanently deprive another of his interest in the property. Therefore, (B) and (C) are incorrect. To be convicted as an accom- plice under the prevailing rule, a person must have acted with the intent to aid or encourage the principal in the commission of the crime charged. Absent a statute, most courts would hold that mere knowledge that a crime may result from the aid provided is insufficient for accomplice liability, at least where the aid involves the sale of ordinary goods at ordinary prices. Here, the proprietor's sale of the crowbar in the ordinary course of business would not make her liable as an accomplice under the prevailing rule even if she believed that the purchaser was going to use it to break into someone's storage unit. Hence, (A) and (C) are incorrect.

HYPO 2A Seeking to track a suspected drug dealer to his hidden lair, the police place a tracking device on a canister of chemicals used for drug production and arrange to have an informant give the suspect the canister. They then follow the suspect to his lair, using the tracking device. Did the police SEIZE the canister by placing the tracker on it?

No because suspect retains full possession of canister, not a seizure under 4th Amendment.

HYPO 4F Police are lawfully on premises and view stereo turntable; they move turntable to view its serial number; they then phone in the numbers to determine if the turntable is stolen. Valid?

No because the criminality of the object is not immediately apparent.

SPECIAL RULES FOR WIRETAPPING AND EAVESDROPPING b. "Unreliable ear."

No warrant is required if one of the parties to the conversation has agreed to the recording. The other party is deemed to have as- sumed the risk that the people to whom he speaks might reveal their conversations to others.

Police obtain a warrant to search the defendant's home for a stolen elephant. In the course of the search, they look in the defendant's dresser drawer. Does this comply with the terms of the warrant?

No, an elephant will not fit into a dresser drawer.

HYPO 3B Police suspect Knotts of being a narcotics trafficker, so they put a beeper tracking device on a canister of chemicals and arrange to have an informant to give the canister to Knotts. Knotts puts the canister in his car and the police track it on public roads. Is this a search within the meaning of the Fourth Amendment?

No, because canister did not belong to Knotts before the tracking device was on it.

Actual Authority: consenting party has actual do- minion or control over premises or object (even if jointly held). Ex.: A child consents to search of family home. Valid?

No, because child doesn't have dominion or control over the house.

HYPO 5A Police use jailhouse "plant" to discuss Defendant's burglary case and obtain confession from Defendant about the burglary after Defendant is arraigned for the burglary. Can the confession be used in the government's case-in-chief for the burglary charge? If Defendant is arraigned on a burglary charge and invokes his right to counsel, can the police later question Defendant regarding his suspected involvement in a hit-and-run?

No, because he's been arraigned, government sent agent in violates 6th amendment Yes, 6th amendment right to counsel is crime specific

HYPO 4D Police falsely state that they have a lawful warrant to search the premises; Defendant acquiesces. Valid? Police seek consent but do not inform Defendant that he has a right to refuse consent. Valid?

No, because police coercing him into consenting. Yes, because when ask someone if they will consent they are normally aware they could say no.

HYPO 2D Police in Los Angeles reasonably suspect that Gary Gangster is carrying narcotics in a briefcase on a flight to New York's LaGuardia airport and call New York Police to inform them. Six hours later, when the plane arrives, they ask Gangster for consent to search his briefcase. When he refuses, they tell him he will have to wait 90 minutes until drug-sniffing dogs can be brought over from Kennedy airport, or leave the bag with them. Valid Terry stop?

No, because police were not diligent -- they had 6 hours notice and they didn't have drug-sniffing dogs. Subsequent delay was unnecessary and not a valid Terry stop.

Special Case of Dissenting Party: consent is invalid if co-owner/occupier is present and objects to search. Police want to search a house and both wife and husband come to door - wife, yes; husband, no. come into house anyway, can they use evidence against husband?

No, but if wait until husband gone and wife says yes, they can search and use.

EXAMPLE Defendant is arrested without probable cause and then is freed on bail. One day later, after consulting with counsel, he voluntarily returns to the police station and confesses to the crime for which he was arrested.

Normally if arrest without probable cause, confession part of fruit of poisonous tree; however, because of gap in time, released from custody - link between illegal police conduct and confession is attenuated.

"SPECIAL NEEDS" SEARCHES b. DNA tests of arrestees.

Police are permitted to take and analyze cheek swabs of arrestees as part of a legitimate booking procedure, where the arrest is for a serious offense and based on probable cause.

Special rule for searches incident to an arrest for driving while intoxicated:

Police may administer a warrantless breathalyzer test, but not a warrantless blood test because a blood test is too intrusive to administer without a warrant.

SEARCH INCIDENT TO ARREST - EXCEPTION:

Police may not search the contents of defendant's cell phone as part of a search incident to arrest. Because smart phones contain such a large quantity of personal information, police must get a warrant before searching them (unless the search is justified by exigent circumstances). Special rule for searches of cars incident to arrest: i. Physical scope is limited to passenger cabin ii. Police can search interior only if: defendant is unsecured (not totally in your control) or they have reason to believe it contains evidence of the crime for which defendant is arrested.

"SPECIAL NEEDS" SEARCHES a. Inventories. to protect you against having property stolen and against you from saying you stole their property

Requires that standardized departmental policy exists and that it be followed.

4. When Exclusion Does Not Occur: a. When state procedural limits on an arrest or search are violated. b. When police, acting with a search warrant, violate the Fourth Amendment's "knock and an- nounce" requirement. c. When there exists an "independent source:" police can show that the evidence which is the fruit of an illegal search was also discovered through a source independent of the improper police action.

State law

SEPARATE SOVEREIGNS a. Double jeopardy does not apply to trials by separate sovereigns. Separate sovereigns can prosecute Defendant for the same offense. Ex.: The states are separate from each other. The U.S. government and the states are separate. A state and its municipalities are not separate.

Tallahassee and Florid -, not separate sovereign.

The probable cause showing may be anticipatory.

That is, the warrant may be based on a showing of probable cause that seizable material will be located on the premises on the date when the warrant will be executed, even though it is not present when the warrant is issued.

PRIOR ACT EVIDENCE

The Due Process Clause permits the use of prior act evidence in criminal cases if it is probative and relevant.

BURDEN OF PROOF CRIMINAL CASE

The Due Process Clause requires that the prosecution in a criminal case prove guilt beyond a reasonable doubt.

"SPECIAL NEEDS" SEARCHES c. Random drug testing.

The Supreme Court has approved random drug tests in a variety of contexts, including: *railroad employees, following an accident. *customs officials who are responsible for drug interdiction. *and students who participate in any extracurricular activity.

A shopper at a flea market noticed a vaseand asked a nearby person how much it cost. The person replied, "One hundred dollars." The shopper paid him the money and took the vase. The person, who was not the owner of the vase but merely a bystander, absconded with the $100. What crime at common law has the bystander committed with respect to the $100? (A) Embezzlement. (B) Larceny by trick. (C) Obtaining property by false pretenses. (D) Nocrime.

The bystander has obtained property by false pretenses. In the crime of false pretenses, the defen- dant obtains title to the property by means of a false representation of a material present or past fact that causes the victim to pass title to his property to the defendant, who knows his representa- tions to be false and intends thereby to defraud the victim. Under the circumstances, by replying that the price of the vase was $100, the bystander implied that he was its owner or was authorized to sell it, which he was not. This false representation induced the victim to pass title to his money to the bystander, who knew the representation to be false. Therefore, he is guilty of obtaining money by false pretenses and (D) is incorrect. (A) is incorrect because the bystander took title, not mere possession. Therefore, the crime is false pretenses, not embezzlement. The physical transfer of that cash without any limitations on its use was a transfer of title to the money. Therefore, the crime is the obtaining of money by false pretenses. (B) is incorrect because larceny by trick occurs when the defendant obtains possession of another's property by lying or trickery. Here, the bystander obtained title to, rather than mere possession of, the money, and is therefore not guilty of larceny by trick.

Acting on a hunch, a police officer went to a young woman's apartment, broke in, and searched it. The officer found exactly what she was looking for under the woman's bed: a sack filled with jewels. The attached note read, "Sweetheart, here are the goods from the estate heist. Your loving boyfriend." It was well known in the community that the woman's boyfriend was a jewel thief. The officer also knew that the estate of a local socialite had been burglarized three days ago. Just as the officer finished reading the note, the woman returned. The officer immediately placed the woman under arrest as an accessory to the estate burglary. Based on the evidence obtained from the woman's apartment, a search warrant was issued for her boyfriend's apartment. The search yielded burglar tools and more jewels from the estate. The boyfriend was immediately arrested and charged with the estate burglary. At the boyfriend's trial for the estate burglary, his attorney files a motion to suppress the evidence consisting of the bag of jewels and note, the tools, and the jewels from the boyfriend's apartment. How should the court rule on the motion? A) Grant the motion as to the bag of jewels and note, but deny it as to the evidence found in the boyfriend's apartment. B) Grant the motion, because all of this evidence is fruit of the poisonous tree. C) Deny the motion, because the police would have caught the boyfriend with the goods eventually. D) Deny the motion, because the police had a warrant to search the boyfriend's apartment.

The court should deny the motion to suppress because the police had a warrant to search the boyfriend's home. The boyfriend's expectation of privacy extended only to his own home, which was searched under a warrant. He does not have standing to assert a Fourth Amendment claim regarding the search of his girlfriend's apartment because her apartment was not his home, and he did not own it or have a right to possession of it. Thus, (A) is incorrect. Because the boyfriend cannot object to the search that provided the probable cause for the search of his apartment, (B) is also incorrect. (C) is not a valid justification because there is nothing to indicate that the seizure would fall under the "inevitable discovery" exception to the exclusionary rule.

INVESTIGATIVE DETENTIONS ("TERRY STOPS") 1. Rule:

The police may detain a person or seize property for investigative purposes if they have a reasonable suspicion of criminal activity or involvement in a completed crime supported by articulable facts. This is a "stop."

In-Home Arrests:

The police must have an arrest warrant to arrest Defendant in his home. a. Exceptions: i. If there is an exigency a. risk that defendant will escape (hot pursuit of fleeing felon) b. danger to police or any person c. risk that defendant will destroy evidence ii. If there is consent (defendant agrees to come into home)

1. Was search done by a government agent?

There are two important categories of government agents for the bar: a. police, on or off duty; and b. private citizens, only if they are acting at the direction of the police. Note: Private security guards are government agents only if they are deputized with the power to arrest. If the answer to Question 1 is yes, go to Question 2.

EVIDENTIARY SEARCH AND SEIZURE

Under the Fourth Amendment, people have a right "to be secure in their persons, houses, papers, and effects, against unreasonable searches and seizures;" warrants must be supported by "probable cause" and "particularly" describe "the place to be searched, and the persons or things to be seized."

Involuntary Manslaughter UKRDCNG

Unintentional killing that results from defendant's criminally negligent conduct

Live witness testimony will not necessarily be excluded, even where the witness's identity was discovered through unlawful police conduct. In determining whether such testimony will be excluded, courts consider the extent to which the witness is freely willing to testify and the extent to which excluding the witness's testimony would deter future illegal conduct.

Witnesses come forward anyway

Third party consent Spouse say ok? Valid?

Yes

HYPO 4E Defendant consents to search of backpack and police enter zipped compartment of backpack and find drugs. Can drugs be basis for prosecution? Defendant consents to search of car and police break open locked briefcase they find in trunk, and find drugs. Can drugs be basis for prosecution?

Yes No, because a reasonable person would not assume reasonable search of car break open locks on my items.

HYPO 3A The police suspect Jones of being a narcotics trafficker, and so they place a GPS tracking device on the bumper of his car while it is parked on a public road, and they use this to track his movements on a public road. Is this a search within the meaning of the Fourth Amendment?

Yes because his car is an effect when police physically intrude on car, search within meaning of 4th Amendment

HYPO 8B Defendant is given use and derivative use immunity and confesses to involvement in a murder. Defendant's co-defendant independently provides prosecutors with a statement outlining Defendant's involvement in a murder the two Defendants allegedly committed. Can Defendant be prosecuted for the murder based on co-defendant's statement?

Yes because immunity grant simply says won't use against defendant in trial

Apparent Authority: consent is valid if police make reasonable factual mistake as to consenting party's authority. Ex.: Police reasonably believe person to be common law spouse. Valid?

Yes because it was a reasonable factual mistake. Woman consents police to search our apartment, which she opens with key, but person no longer with person who's apartment it was, no authority to consent to search.

Police pull over Cleveland and develop probable cause to believe he's hiding drugs in the car. They seize the car and tow it to the police impound lot, where they search the car. Valid search?

Yes because no requirement automobile search be contemporaneous or place

HYPO 5B Defendant was arrested, provided Miranda warnings, and indicated that she wished to talk to her lawyer. She was told that she could do so after her fingerprints were taken. While being fingerprinted, however, Defendant blurted out that "getting a lawyer is a waste of time, because I know you got the goods on me." Can the prosecution use the statement in its case- in-chief?

Yes because she just spoke and wasn't being questioned, so no Miranda violation.

HYPO 3D Police have probable cause to believe that Clyde Culprit has narcotics somewhere in his car, and, therefore, have the right to perform a warrantless search of Clyde's car (see below). In the course of this search, do they have the right to search the suitcase of Andy accomplice, Clyde's passenger, even though they have no probable cause relating to Andy?

Yes because suitcase is considered part of the car and can search as part of their car search. Can't search Andy, but can search his suitcase.

HYPO 6B Police wrongfully interrogate Defendant and obtain a confession about a murder. As a result, a contemporaneous search for the body of the victim is ended. While police cannot use the confession in their case-in-chief, can they use information on the location and condition of the body?

Yes because that search party would have ultimately found the body, so body can be used in evidence at trial.

HYPO 7A The jury venire serving as the basis for the petit jury used to convict Defendant of burglary was devoid of females. Can Defendant raise a Sixth Amendment challenge?

Yes because women excluded from venire

HYPO 3C Police suspect Jardines of selling drugs out of his house, so they take a drug-sniffing dog up the walkway in Jardines' yard to the front door, where the dog alerts for drugs. Search?

Yes, because although they have limited license to go into curtilage, the use of drug-sniffing dog exceeds the scope of this license.

HYPO 8A Defendant is driving his car excessively fast on a residential street and hits a small child, killing him, and then drives away at top speed. He is tried for manslaughter and acquitted. Can he subsequently be tried for leaving the scene of an accident?

Yes, because each offense contains an element that the other does not.

HYPO 4A Police knock loudly on the door of an apartment they have probable cause to believe contains illegal narcotics. In response to the knock they hear shuffling and voices that lead them to believe the narcotics are about to be destroyed. May they enter the apartment without a warrant even though it was their knocking that created the risk of evidence destruction?

Yes, because knocking neither violated nor threatened to violate the 4th amendment

HYPO 4B Police have probable cause to believe that Carney keeps marijuana in his mobile home to give teenagers in exchange for sex. Carney lives in the mobile home, which is parked in a public lot. Police search the mobile home without a warrant. Valid search?

Yes, his mobile home is a vehicle

HYPO 5D Police extract Miranda-defective murder confession and information on location of gun used in murder. Is the gun, with fingerprints on it, admissible?

Yes, the gun is admissible even if never found it if violated Miranda rights -- Miranda does not apply fruit of poisonous tree

HYPO 4G Police lawfully stop Defendant and notice a bulge in his jacket. During a frisk police immediately feel what they know to be a foil-wrapped "brick" of drugs. Can they use the drugs in a prosecution?

Yes, under the Plain Feel doctrine

Stop and frisk TS RBAD PDO MSAPF

a) During valid Terry stop (see above) b) Police have reasonable belief that detainee is armed and dangerous c) May pat down outer clothing for weapons d) May seize anything that by plain feel is weapon or contraband

Consent (from one with apparent right to use or occupy property) SO PCR

a) If suspect present, may overrule consent b) Parent usually has authority to consent to search of child's room if parent has access

AUTOMOBILE EXCEPTION

a. Standard: Police need probable cause to believe that evidence or contraband will be found in the car. b. Scope of search: Police may search the entire car and they may open any package, luggage, or other container that may reasonably contain the items for which there is probable cause to search.

INVESTIGATIVE DETENTIONS ("TERRY STOPS") 2. Scope:

a. The stop must be no longer than necessary to conduct a limited investigation to verify or dispel the suspicion. b. The police must be diligent in pursuing this investigation. c. The police may ask persons to identify themselves. d. The police may arrest suspect for failure to comply. e. The stop may turn into an arrest if the police develop probable cause that defendant committed some crime. f. The police can "frisk" (Terry "frisk") for weapons if they have reasonable suspicion that the defendant is armed and dangerous.

WHAT IS THE SAME OFFENSE?

a. Two crimes are not the same offense if each re- quires proof of an element that the other does not.

PLAIN VIEW Items that are in "plain view" can be seized without a warrant if: a. the police have lawful _______ to the vantage point from which they see the items; b. the police have _______ access to the items themselves; c. the criminality of the object is __________ apparent.

a. access b. lawful c. immediately

Right to Terminate Interrogation a. Rule: Defendant may terminate custodial interrogation at any time by invoking either the right to remain silent or the right to counsel. The invocation must be __________ and clear. b. If Defendant invokes his right to remain silent, police must _______ the request and cease interrogation. Police can reinitiate questioning if they wait a significant period of time and first obtain a Miranda waiver. c. If Defendant invokes his right to counsel, police must honor the request and ______ interrogation. Police cannot thereafter reapproach Defendant on their own. (magic police cannot come back) They can interrogate only if: i. counsel is present; or ii. defendant reinitiates the interrogation; iii. defendant is released from custody for 14 days. d. Fifth Amendment right to counsel is not offense-specific; it bars questioning as to _____ other suspected crimes.

a. explicit b. honor c. cease d. all

GRAND JURIES a. Only the _________ government must use grand juries. Many states do use grand juries, but they are not required to do so under the U.S. Constitution. Many states allow prosecutors to charge by information. b. The ___________ rule does not apply in the grand jury context; a grand jury witness may be compelled to testify about illegally seized evidence. c. Grand juries are conducted in __________. The accused has no right to appear and has no right to have counsel present or to have witnesses testify.

a. federal b. exclusionary c. secret

EXCEPTIONS PERMITTING RETRIAL AFTER JEOPARDY HAS ATTACHED a. ______ jury, which means the failure to reach a verdict of guilty or not guilty. Note: The mere fact that an otherwise hung jury agrees that a defendant is not guilty of some charges does not prevent retrial on those charges if the jury never actually issues a verdict. b. manifest _________ c. __________ appeal However, retrial is not permitted if reversal was based on insufficiency of the evidence.

a. hung b. necessity. some deep reason of justice need to permit new trial c. successful.

Uses for Miranda-defective Statements

a. impeachment of Defendant (but not third party witnesses) b. discovery of physical evidence or witnesses c. public safety exception

DOUBLE JEOPARDY a. Jeopardy attaches in a jury trial when the _____ is empanelled. b. Jeopardy attaches in a bench trial when the first _______ is sworn. c. Jeopardy attaches in a guilty plea when the court ___________ accepts the plea. d. Jeopardy does not attach in _______ cases.

a. jury b. witness c. unconditionally d. civil

THE RIGHT TO CONFRONT WITNESSES a. The defendant in a criminal ___________ has a Sixth Amendment right to confront adverse witnesses. This right seeks to ensure that: i. The fact finder and the defendant observe the demeanor of the testifying witness; and ii. The defendant has the opportunity to cross-examine any witness testifying against him. b. Absence of a face-to-face confrontation between the defendant and the accuser does not __________ the Sixth Amendment when preventing such confrontation serves an important public purpose (such as insulating a child witness from trauma) and the reliability of the witness's testimony is other- wise assured.

a. prosecution b. violate

7.1 THE RIGHT TO TRIAL BY JURY a. A right to a jury trial exists when the possible sentence for an offense is more than _____ months. b. There is no constitutional right to a jury of 12. The minimum number of jurors is ____. c. Non-unanimous _____ verdicts. i. The Court has upheld convictions that were less than unanimous: 10-2 or 9-3, but would not likely uphold an 8-4 verdict. ii. Any 6-member jury must be unanimous. d. Jury _____________ i. Defendant has a right to have a jury selected from a representative cross-section of the community. This pool of individuals is known as the venire. ii. Defendant does not have the right to proportional representation of all groups on his particular petit jury (as opposed to the venire from which it is drawn). e. Limits on Peremptory Challenges: ______ and Gender i. It is unconstitutional for either the prosecution or the defense to exercise its peremptory challenges to exclude jurors solely on account of race or gender.

a. six b. six c. jury d. Selection e. Race

Enforcing the Exclusionary Rule a. Applicability of the rule is decided by a judge, outside hearing of jury. b. Defendant has a right to testify at a suppression hearing without his testimony being admitted against him on the issue of guilt. c. The prosecution bears the burden of establishing admissibility of evidence, by a ___________ of the evidence. d. Exclusion of tainted evidence is not ________; it depends on the reasonableness of the police conduct and likelihood that exclusion will deter such conduct.

a. suppression hearing and jury will not hear b. will not be used against him in trial c. preponderance d. automatic

SHOCKING METHODS Evidence obtained through methods that "shock the conscience" is inadmissible under the due process clause. a. Blood tests do not shock the conscience because they involve almost no pain or risk, but invasive procedures like _________ to recover a bullet for evidence are impermissible because they involve serious risks and a severe intrusion on privacy, at least where other evidence is available. b. "Shocking inducement." If a crime is induced by official conduct that shocks the conscience, any subsequent conviction _________ due process.

a. surgery b. violates

Waiver of Miranda: a. A suspect may waive Miranda warnings, but the prosecution must prove, based on a __________ of the circumstances, that the waiver was: i. knowing; ii. intelligent; and iii. voluntary. b. No waiver is __________ when a suspect is silent or shrugs his shoulders. However, if warnings are given and suspect answers questions, then waiver is inferred. c. Defendant need not be told of ____ subjects that will be covered in the interrogation. d. Police can fail to inform Defendant that a ______ party has retained a lawyer for Defendant.

a. totality b. inferred c. all (my cousin Vinny, can of beans, but interrogated for murder) d. third (even when lawyer ready willing and able to defend)

CONSENT a. To be valid, consent must be knowing and ___________. b. Scope of consent: the search is limited to areas in which a ___________ person would believe it extends.

a. voluntary b. reasonable

SEARCH INCIDENT TO ARREST a. Arrest must be lawful under the Fourth Amendment. b. Justifications for search: ______________ c. Time and place of search: ______________ d. Scope of search: ________________

b. Officer safety and pres- ervation of evidence. c. Search must be contemporaneous in time and place with the arrest. d. Only areas within defendant's immediate control. Includes defendant's person, clothes, and any containers within his "grab zone."

B A R R K

burglary arson robbery rape kidnapping

Good faith exception: Illegally obtained evidence will not be excluded at trial when police acted in "good faith" based on: a. Judicial approval of search warrant b. Subsequently invalidated statute c. Erroneous information in arrest warrant d. Supreme Court ruling that is subsequently overturned. e. A reasonable but mistaken belief that defendant's conduct violates the law.

c. arrest warrant withdrawn by court, police system doesn't reflect that e. person driving with one headlight out - honest, but mistake belief violated NC law; even though officer wrong about that fact, officer acting in reasonable good faith belief violating law

EXAMPLE Counsel fails to timely file a notice of appeal, or, in capital trial, fails to investigate Defendant's life history with respect to potential mitigating circumstances for jury's consideration.

conduct falling below objective standard of reasonableness and likely changed outcome = ineffective assistance of counsel

Under the M'Naghten rule, a defendant is entitled to an acquittal if the proof establishes that a disease of the mind caused a _______ of reason, such that the defendant lacked the ability at the time of his actions to either know the wrongfulness of his actions or understand the nature and quality of his actions.

defect

FIFTH AMENDMENT d. Grant of Immunity

i. A witness may be compelled to answer an incriminating question if granted immunity from prosecution. ii. The most common form of immunity is use and derivative use immunity. It guarantees both that the witness's testimony and evidence secured by means of the testimony will not be used against the witness. iii. However, witnesses may still be prosecuted if the government shows that the evidence to be used against them was derived from a source independent of the immunized testimony, or obtained before the grant of immunity.

FIFTH AMENDMENT PRIVILEGE AGAINST SELF-INCRIMINATION a. Who May Assert the Privilege?

i. Any natural person asked questions under oath is entitled to assert the privilege in any criminal trial, civil trial, or hearing. ii. One must assert the privilege, and not answer, or it will be waived for purposes of all subsequent proceedings.

FIFTH AMENDMENT c. Comments on Defendant's Silence

i. It is unconstitutional for the prosecutor in a trial to comment on a defendant's failure to testify. ii. A prosecutor may not comment on defendant's silence after being arrested and informed of his Miranda rights. iii. At trial Defendant may ask the court for a jury instruction that tells the jury they may draw no adverse inference from his failure to testify.

FIFTH AMENDMENT: b. Scope of the Privilege

i. The Fifth Amendment protects only testimony.

If the Warrant Is Invalid If the warrant is invalid (e.g., lacking in probable cause), did the officer's "objective good faith" reli- ance on the warrant redeem the search? NOTE: The good faith exception is not applicable when: i. it's based on a bare bones ________; ii. it does not satisfy the __________ requirement; iii. the affiant knowingly or ________ includes false information in the affidavit; or iv. the magistrate is ________.

i. affidavit ii. particularity iii. recklessly iv. biased

3. Does the defendant have a reasonable expec- tation of privacy in the place searched? b. Unprotected Areas

i. bank accounts ii. curtilage viewed from public airspace iii. garbage on curb iv. voice exemplars v. odors (dog sniffs of car or luggage) vi. handwriting vii. open fields (any outdoor area that's not part of the curtilage) viii. paint on exterior of car Note: What do these all have in common? They are all exposed to the view of third parties.

Home of Third Party: Police cannot search for the subject of an arrest warrant in the home of a third party. Police must either:

i. obtain a search warrant for the subject, specifying his location in the third party's home; ii. secure consent to search the premises from the third party homeowner; or iii. enter on the basis of exigent circumstances.

2. Does the defendant have a reasonable expec- tation of privacy in the place searched? a. Protected Areas:

i. person ii. house - including curtilage (the area surrounding house - domestic yard area) iii. papers iv. effects (personal property)

Was Warrant Supported by Probable Cause? i. Test: Based on the______________, whether there is a fair probability that the area searched contains evidence of a crime. ii. Use of Informants: A warrant can be based on an informant's tip even if the informer is ___________. The affidavit may be sufficient even though either the reliability or credibility of the informer or her basis of knowledge is not established. The informer's identity generally need not be revealed.

i. totality of the circumstances ii. anonymous

Execution of the Warrant i. Did police comply with terms of warrant? ii. Did police wait too long to execute warrant? ("staleness") iii. Did police violate "knock and announce" rule? (not a ground for excluding evidence - see below)

iii. may violate 4th amendment, but not ground for excluding evidence.

When police would have "inevitably discovered" the evidence.

no exclusion - hypothetical independent source doctrine

EXAMPLE A court of appeals reverses Defendant's burglary conviction because the evidence did not support guilt beyond a reasonable doubt. A retrial would violate the double jeopardy clause.

not enough evidence, cannot do, but erroneous jury instruction, ok

STOP AND FRISK c. Special Rules for Autos:

police can "frisk" interior if reasonable belief that weapon is present; police can order both driver and any passengers out of the car—even without suspicion that they are armed; anyone in car can be frisked, if reasonable suspicion exists that they are armed.

EXAMPLE Defendant is arrested in store as suspect in armed robbery. Police ask, "Where is the gun?" Defendant replies, "I stashed it over there," and leads police to gun. Both Defendant's statement and gun admissible in case-in-chief.

questioning for public safety, so okay, statement and gun admissible

STOP AND FRISK a. Police must have reasonable suspicion that defendant is armed and dangerous. Purpose is solely for ______, not discovery of evidence. b. Scope: limited to exterior of clothing, except that police can reach directly into area of clothing if they have specific ________ information that weapon is concealed there.

safety reliable

Defendant Can Refuse to Take Stand Altogether 1. Prosecutor cannot comment on defendant's ________ after receiving Miranda warnings Exception—in response to a claim of no opportunity to explain Harmless error test applies—violation does not automatically require retrial

silence

Sixth Amendment right to jury for serious offenses Serious offense—imprisonment for more than ______months Civil contempt—no right

six

As to an affirmative defense to a criminal charge, the Supreme Court has held that the _______ can place the burden of proof on the _____ without violating the defendant's constitutional rights.

state defendant

EXAMPLE Where members of a state legislative commission inform the defendant that he has a privilege against answering their questions, he cannot later be convicted for refusing to answer their questions.

this is too shocking

For purposes of the Fourth Amendment, a seizure occurs when a reasonable person would believe that he is not free to leave. The courts consider the ____________ of the circumstances in making this determination.

totality

In-court identification. A court will not exclude an in-court identification simply because it is the fruit of a prior ________ detention.

unlawful

The United States Supreme Court has held that a person does not have a reasonable expectation of privacy in any land or field not a part of the curtilage. Thus, there is no Fourth Amendment protection in such areas. Therefore, the police did not ________ the Fourth Amendment when they took pictures or when they cut through the fence and entered the field.

violate


Conjuntos de estudio relacionados

Geometry and Measurement - Geometric Shapes - Quiz 2

View Set

Learning Module 34: Reaction Stoichiometry Basics

View Set

Ch 7: Legal Dimensions of Nursing Practice (NUR 111-nursing fundamentals book)

View Set

Unit 5- Chapter 51-Clients with DM

View Set